Final Exam (Part 1)

Ace your homework & exams now with Quizwiz!

When assessing the skin, nurses use techniques to provide complete data and correct documentation. Which actions are appropriate during the skin assessment? Select all that apply. A. Comparing bilateral parts for symmetry B. Proceeding in a toe-to-head, systematic manner C. Using standard terminology to communicate and document findings D. Avoiding using data from the nursing history to direct the assessment E. Documenting only skin abnormalities on the health record F. When risk factors are identified, following up with a related skin assessment

a, b, c, f. During skin assessment, the nurse should compare bilateral parts for symmetry, use standard terminology to communicate and document findings, and perform the appropriate skin assessment when risk factors are identified. The nurse should proceed in a head-to-toe systematic manner, using cues/data from the nursing history to direct the assessment. When documenting a physical assessment of the skin, the nurse should describe exactly what is observed or palpated, including appearance, texture, size, location or distribution, and characteristics of any findings.

A nursing student asks an experienced nurse why they provide massage for their patients. Which of these would be reflected in the nurse's response? A. To help with pain management B. To provide comfort C. To communicate to patients through touch D. To energize patients, especially those with dementia E. To facilitate healing after back or spinal surgery F. To help increase circulation

a, b, c, f. The benefits of massage include general relaxation and increased circulation, pain relief, sleep promotion, and increased patient comfort and well-being. Massage also provides an opportunity for the nurse to communicate and connect with the patient through touch. Back massage is contraindicated if the patient has had back surgery or has fractured ribs.

A nurse working in a rehabilitation facility focuses on the goal of restoring health for patients. Which nursing interventions could the nurse use to meet this goal? Select all that apply. A. Counseling adolescents in a drug rehabilitation program B. Performing range-of-motion exercises for a patient on bedrest C. Demonstrating insulin injection to a patient with diabetes D. Recommending a yoga class for a busy executive E. Providing hospice care for a patient with end-stage cancer F. Teaching a nutrition class at a local high school

a, b, c. Activities to restore health focus on the person with an illness and range from early detection of a disease to rehabilitation and teaching during recovery. These activities include drug counseling, teaching patients how to administer their medications, and performing range-of-motion exercises for bedridden patients. Recommending a yoga class for stress reduction helps prevent illness, and teaching a nutrition class helps promote health. A hospice nurse facilitates coping with disability and death.

A nurse caring for patients in a critical care unit knows that providing good oral hygiene is an essential to good patient outcomes, especially for those receiving mechanical ventilation. What are positive outcomes expected from this care? Select all that apply. A. Promoting the patient's sense of well-being B. Preventing deterioration of the oral cavity C. Contributing to decreased incidence of aspiration pneumonia D. Eliminating the need for flossing E. Decreasing oropharyngeal secretions F. Compensating for an inadequate diet

a, b, c. Adequate oral hygiene is essential for promoting the patient's sense of well-being and preventing deterioration of the oral cavity. Diligent oral hygiene and use of chlorhexidine gluconate (CHG) in critical care areas, can limit the growth of pathogens in oropharyngeal secretions, decreasing the incidence of ventilator-associated pneumonia, aspiration pneumonia, and other systemic diseases. Oral care does not eliminate the need for flossing, decrease oropharyngeal secretions, or compensate for poor nutrition.

The nurse coach at a cardiac rehabilitation office is meeting with a patient who has learned they have heart failure. Which nursing actions might the nurse coach include in coaching sessions for this patient? (Select all that apply.) A. Provide education based on the patient's personal goals. B. Explore the patient's readiness for change. C. Assist the patient to determine progress toward goals. D. Direct the patient to exercise daily. E. Identify goals for the patient.

a, b, c. The nurse coach facilitates change or development that assists the individual to cope with health challenges. The nurse coach establishes a partnership with the patient to support the patient to identify and work toward the patient's personal agenda and goals; nurse coaches do not use teaching and other strategies directed by the nurse as an expert. A, b, and c are patient-driven, person-centered interventions to educate and empower the patient. D and e are interventions identified and directed by the nurse, not by the patient, which is not part of the coaching process.

A nurse is caring for a patient with a painful, non-healing surgical wound. The patient does not request pain medication because they do not want to be a burden. What actions will the nurse implement to improve pain relief? Select all that apply. A. Reestablishing the pain level the patient finds acceptable as the pain management goal B. Obtaining a dry-erase board to remind the patient of the plan of care C. Assessing the patient's pain and offering analgesia during hourly rounding D. Placing the analgesic underneath other medications and quickly handing it to the patient E. Asking the family members to speak to the patient about pain relief

a, b, c. The nurse reassesses the patient's knowledge and acceptance of the plan of care, including the level of pain the patient finds acceptable. Using a dry-erase board and hourly rounding further communicate and reinforce the care plan. The nurse develops a compassionate and trusting relationship with the patient; the nurse and patient mutually determine the plan for pain management, not their family

A patient is admitted with a nonhealing surgical wound. Which nursing interventions will the nurse use to promote wound healing? Select all that apply. A. Applying sterile dressing supplies B. Discussing zinc supplementation with the health care provider C. Maintaining bedrest D. Performing careful hand hygiene E. Teaching the patient to increase protein in the diet F. Suggesting the patient consume vitamin C-containing foods.

a, b, d, e, f. Careful hand washing (medical asepsis) is the most important. The nurse will use sterile dressings and supplies and promote intake of vitamins, zinc, and protein. Depending on the site of the wound and condition of the patient, bedrest may be indicated.

A nurse is preparing medications for patients in the ICU. The nurse is aware that patient variables may affect the absorption of these medications. Which statements accurately describe these variables the nurse will use as a basis for practice? Select all that apply. A. Patients in certain ethnic groups obtain therapeutic responses at lower doses or higher doses than those usually prescribed. B. Some people experience the same response with a placebo as with the active drug used in studies. C. People with liver disease metabolize drugs more quickly than people with normal liver functioning. D. A patient who receives a pain medication in a noisy environment may not receive full benefit from the medication's effects. E. Oral medications should not be given with food as the food may delay the absorption of the medications. F. Circadian rhythms and cycles may influence drug action.

a, b, d, f. Nurses are responsible for applying pharmacologic knowledge during medication administration, including varied responses in patients from different ethnic groups (ethnopharmacology). The patient's expectations of the medication may affect the response to the medication; for example, when a placebo is given and a patient has a therapeutic effect. Environment may also influence the patient's response to medications; for example, sensory deprivation and overload may affect drug responses. Circadian rhythms and cycles may also influence drug action. The liver is the primary organ for drug breakdown, thus pathologic conditions that involve the liver may slow metabolism and alter the dosage of the drug needed to reach a therapeutic level. The presence of food in the stomach can delay the absorption of orally administered medications. Alternatively, some medications should be given with food to prevent gastric irritation, which the nurse should consider when establishing a patient's medication schedule. Other medications may have enhanced absorption if taken with certain foods.

The nurse is cleaning an open abdominal wound that has edges that are not approximated. What are accurate steps in this procedure? Select all that apply. A. Use standard precautions or transmission-based precautions when indicated. B. Moisten a sterile gauze pad or swab with the prescribed cleansing agent and squeeze out excess solution. C. Clean the wound in full or half circles beginning on the outside and working toward the center. D. Work outward from the incision in lines that are parallel to it from the dirty area to the clean area. E. Clean to at least 1 inch beyond the end of the new dressing if one is being applied. F. Clean to at least 3 inches beyond the wound if a new dressing is not being applied.

a, b, e. The correct procedure for cleaning an open wound with edges that are not approximated is: (1) use standard precautions and appropriate transmission-based precautions when indicated, (2) moisten sterile gauze pad or swab with prescribed cleansing agent and squeeze out excess solution, (3) use a new swab or gauze for each circle, (4) clean the wound in full or half circles beginning in the center and working toward the outside, (5) clean to at least 1 inch beyond the end of the new dressing, and (6) clean to at least 2 inches beyond the wound margins if a dressing is not being applied.

A nursing student is committed to providing thoughtful, person-centered care. Which nursing actions demonstrate this type of care? Select all that apply. A. Assisting patients to select meals based on their cultural observances B. Providing nursing care based on patients' needs and preferences C. Documenting nursing interventions in the electronic health record D. Reviewing fingerstick blood glucose levels with the primary nurse E. Listening to a patient's concern for their ill significant other

a, b, e. The nursing process ensures that nurses are person centered rather than task centered. Attending to cultural preferences and needs and listening to a patient's concerns are patient-centered actions. Documentation and communication with other members of the health care team are not specifically patient centered.

Nursing students enrolled in a medical-surgical nursing course are learning about infection control measures. They have learned that nurses use droplet precautions for patients with which infections? Select all that apply. A. Rubella B. Herpes simplex C. Varicella D. Tuberculosis E. MRSA F. Adenovirus

a, b, f. Rubella, diphtheria, and adenovirus infection are illnesses transmitted by large-droplet particles; droplet precautions and standard precautions are indicated. Airborne precautions refer to small, infectious particles spread through the air; for example, tuberculosis, varicella, and rubeola. Contact precautions are used for patients who are infected or colonized by a multidrug-resistant organism (MDRO), such as MRSA.

According to Nursing: Scope and Standards of Practice (ANA, 2021), "Nursing is the diagnosis and treatment of human response, and advocacy in the care of individuals, families, groups, communities, and populations in recognition of the connection of all humanity. Which actions are consistent with these nursing roles? Select all that apply. A. Offering emotional support to a patient overwhelmed by a new diagnosis of ovarian cancer B. Making a diagnosis of appendicitis when a patient reports right lower quadrant pain C. Providing classes on nutrition and physical activity at a senior center D. Administering an antibiotic to a patient with a documented infection E. Working with the health department to provide free influenza vaccinations to children F. Requesting the health care provider change prescription for the patient whose pain is still unrelieved

a, c, d, e, f. Education, emotional support, nursing interventions such as administering medications, preventing influenza and infectious diseases, and advocating for improved pain management are roles of the nurse. Making a medical diagnosis is not within the scope of professional nursing practice.

A nurse on a surgical unit is concerned about a colleague's possible substance use disorder. Which signs and symptoms could support the nurse's suspicion? Select all that apply. A. Exhibiting diminished alertness and somnolence while working B. Attending multiple continuing education conferences C. Offering to medicate coworkers' patients for pain D. Making incorrect narcotics counts and creating wastage E. Leaving the unit frequently

a, c, d, e. Signs of substance use in nurses may include diminished alertness or somnolence, leaving the unit frequently, incorrect narcotic counts, wastage, offers to medicate colleagues' patients, or changes in job performance, among others. Attending professional conferences is an example of a nurse who is fully engaged with their work.

The nurse collects subjective and objective data during a patient assessment. When documenting, which data points will the nurse include as subjective data? Select all that apply. A. Feeling nauseated B. Edematous ankles C. Feeling anxious about test results D. Report of left arm tingling E. Pain rated 7 on a scale of 1 to 10 F. Oral temperature of 101°F

a, c, d, e. Subjective data are information perceived only by the affected person. Examples of subjective data are feeling nauseated, anxious, tingling, and experiencing pain. Objective data are observable and measurable data that can be seen, heard, felt, or measured by someone other than the person experiencing them. Examples of objective data are fever or 101°F or edema.

A nursing unit has multiple patients with MRSA infections requiring contact isolation. In which situations is it appropriate for the nurses to use an alcohol-based hand sanitizer to decontaminate their hands? Select all that apply. A. Before providing a bed bath B. Having visibly soiled hands after patient contact C. Removing gloves after patient care D. Inserting a urinary catheter E. Assisting with a surgical placement of a cardiac stent F. Removing old magazines from a patient's table

a, c, d, f. An alcohol-based handrub is used in the following situations: before direct contact with patients; after direct contact with patient skin; after contact with body fluids if hands are not visibly soiled; after removing gloves; before inserting urinary catheters, peripheral vascular catheters, or invasive devices not requiring surgical placement; before donning sterile gloves prior to an invasive procedure; when moving from a contaminated body site to a clean body site; and after contact with objects contaminated by the patient. It is essential to note that handrubs are not appropriate for use after caring for a patient with C. diff infection.

A community health nurse is developing a support group for patients coping with chronic health problems. What chronic health conditions does the nurse anticipate group members might want to discuss? Select all that apply. A. Diabetes mellitus B. Bronchial pneumonia C. Rheumatoid arthritis D. Ulcerative colitis E. Fractured hip F. Otitis media

a, c, d. Diabetes, arthritis, and ulcerative colitis are chronic diseases causing permanent, irreversible alterations in normal anatomy and/or physiology. They require a lengthy period of care, support, and patient education. Pneumonia, fractures, and otitis media are acute illnesses, characterized by a rapid onset and typically short duration

A nursing student is learning to perform physical assessments. When will the student use the technique of palpation? Select all that apply. A. Assessing temperature of a patient's feet B. Counting the apical pulse C. Assessing for skin turgor D. Evaluating lymph nodes E. Assessing for dullness over a tumor F. Finding a heart murmur

a, c, d. During palpation, the nurse uses the sense of touch to compare bilateral pulses for symmetrical temperature, assess skin turgor, and check for enlarged lymph nodes. During percussion, the fingertips are used to tap the body over body tissues to produce vibrations and sound waves. The characteristics of the sounds provide information about the location, shape, size, and density of tissues, such as dullness over a mass or fluid accumulation. Auscultation refers to listening with a stethoscope to sounds produced in the body; counting the apical pulse or auscultating heart sounds/murmurs are examples.

The nurse assessing a patient plans to use the OLD CARTS mnemonic to organize their questions. What questions will the nurse include in the assessment? Select all that apply. A. "Can you tell me when the problem began"? B. "Where were you sitting when this started?" C. "Have your symptoms stopped and/or started again?" D. "Would you describe your pain as sharp, dull or burning?" E. "What do you believe has caused this problem?"

a, c, d. The OLD CARTS mnemonic refers to Onset: "When did your symptom(s) begin"?. Location: "Where is the symptom"?. Duration: "Is it episodic or constant?" or "How long does it last"?. Characteristics: "How would you describe it?." Alleviating and Aggravating factors: "What makes it better or worse?." Relieving factors: "What makes it better?." Treatments: "Have you tried anything to make it better?." Severity: "On a scale of 1-10, with 1 being the lower number, how serious is the symptom?."

A nurse has finished providing care for a patient in contact isolation for a MRSA infection. Place the steps the nurse should follow to remove PPE in the correct order. A. Untie gown at the front waist B. Remove mask C. Remove gloves D. Remove gown E. Remove goggles

a, c, e, d, b. If an impervious gown has been tied in front of the body at the waist, the nurse should untie the waist strings before removing gloves. Gloves are removed first because they are most likely to be contaminated, followed by the goggles, gown, and mask. Hands should be washed thoroughly after the equipment has been removed and before leaving the room.

A nurse is attempting to improve care on the pediatric unit of a hospital. Which improvements might the nurse employ when following the recommendations of the Institute of Medicine's Committee on Quality of Health Care in America? Select all that apply. A. Basing patient care on continuous healing relationships B. Customizing care to reflect the competencies of the staff C. Using evidence-based decision making D. Having a charge nurse as the source of control E. Using safety as a system priority F. Recognizing the need for secrecy to protect patient privacy

a, c, e. Care should be based on continuous healing relationships and evidence-based decision making. Customization should be based on patient needs and values with the patient as the source of control. Safety should be used as a system priority, and the need for confidentiality of care with transparency for the patient and designated individuals, rather than secrecy is used.

A nurse on a surgical unit is working with a nursing student and discussing various phases of wound healing for postoperative patients. Which statements accurately describe these stages? Select all that apply. A. Hemostasis occurs immediately after the initial injury. B. A liquid called exudate is formed during the proliferation phase. C. White blood cells move to the wound in the inflammatory phase. D. Granulation tissue forms in the inflammatory phase. E. During the inflammatory phase, the patient has generalized body response. F. A scar forms during the proliferation phase.

a, c, e. Hemostasis occurs immediately after the initial injury, and exudate occurs in this phase as plasma and blood components leak out into the injured area. White blood cells, predominantly leukocytes and macrophages, move to the wound in the inflammatory phase to ingest bacteria and cellular debris. During the inflammatory phase, the patient has a generalized body response, including a mildly elevated temperature, leukocytosis (increased number of white blood cells in the blood), and generalized malaise. New tissue, called granulation tissue, forms the foundation for scar tissue development in the proliferation phase. New collagen continues to be deposited in the maturation phase, which forms a scar.

During orientation to the critical care unit, a nurse learns that staff follow existing clinical practice guidelines, also called standards, for patient care. Which activities does the nurse expect to be included in these guidelines? Select all that apply. A. Monitoring vital signs and pulse oximetry every hour B. Using intuition to troubleshoot patient problems C. Repositioning a patient on bed rest every 2 hours D. Becoming a nurse mentor to a student nurse E. Administering pain medication prescribed by the health care provider F. Becoming involved in community nursing events

a, c, e. Standards are the levels of performance accepted and expected by the nursing staff or other health care team members. They are established by authority, custom, or consent. Standards would include monitoring patient status every hour, repositioning a patient on bed rest every 2 hours, and administering pain medication prescribed by the health care provider. Using intuition to troubleshoot patient problems, becoming a nurse mentor to a student nurse, and becoming involved in community nursing events are not included in patient care standards.

A nurse is using the Johns Hopkins Nursing Evidence-Based Practice (JHNEBP) model PET as a clinical decision-making tool when delivering care to patients. Which steps reflect the intended use of this tool? Select all that apply. A. Recruiting an interprofessional team to develop and refine an EBP question B. Drawing from personal experiences of being a patient to establish a therapeutic relationship with a patient C. Conducting a search of electronic data bases for current treatments for type 2 diabetes D. Drawing on their spiritual training when counseling a patient in hospice care at end of life E. Questioning the protocol for assessing postoperative patients returning to the ICU F. Studying pathophysiology to understand the disease states of assigned patients

a, c, e. The JHNEBP model is a powerful problem-solving approach to clinical decision making, which uses a three-step process called PET: practice question, evidence, and translation. The goal of the model is to ensure that the latest research findings and best practices are quickly incorporated into patient care. Steps in PET include, but are not limited to, recruiting an interprofessional team, developing and refining the EBP question, and conducting internal and external searches for evidence.

A nurse is teaching a patient how to use a meter-dosed inhaler to control asthma. What are appropriate guidelines for this procedure? Select all that apply. A. Shake the inhaler well and remove the mouthpiece covers from the MDI and spacer. B. Take shallow breaths when breathing through the spacer. C. Depress the canister releasing one puff into the spacer and inhale slowly and deeply. D. After inhaling, exhale quickly through pursed lips. E. Wait 1 to 5 minutes as prescribed before administering the next puff. F. Gargle and rinse with salt water after using the MDI.

a, c, e. The correct procedure for using a meter-dosed inhaler is: Shake the inhaler well and remove the mouthpiece cover; breathe normally through the spacer; depress the canister releasing one puff into the spacer and inhale slowly and deeply; after inhaling, hold breath for 5 to 10 seconds, or as long as possible, and then exhale slowly through pursed lips; wait 1 to 5 minutes as prescribed before administering the next puff; and gargle and rinse with tap water after using the MDI.

Nurses on an oncology unit plan to adopt use of critical pathways for patients receiving chemotherapy. What positive features of this system will the nurses anticipate? Select all that apply. A. Accessible computerized practice standards, easily individualized for patients B. Binary decision tree for stepwise assessment and intervention C. Ability to measures the cause-and-effect relationship between pathway and patient outcomes D. Research-based practice recommendations that may or may not have been tested in clinical practice E. Preprinted provider prescriptions, using standards validated through research, to streamline care F. Outcomes with suggested time frames for achievement

a, c, f. Critical pathways represent a sequential, interdisciplinary, minimal practice standard for a specific patient population, that provide flexibility to alter care to meet individualized patient needs. They provide the ability to measure a cause-and-effect relationship between pathway and patient outcomes. An algorithm is a binary decision tree that guides stepwise assessment and intervention with intense specificity and no provider flexibility. Guidelines are broad, research-based practice recommendations that may or may not have been tested in clinical practice, and an order set is a preprinted provider order used to expedite the order process after a practice standard has been validated through analytical research.

A nurse is providing active-assistive range-of-motion exercises for a patient who is recovering from a stroke. During the session, the patient reports that they are "too tired to go on." What actions are appropriate at this time? Select all that apply. A. Stop performing the exercises. B. Decrease the number of repetitions performed. C. Reevaluate the plan of care. D. Move to the patient's other side to perform exercises. E. Encourage the patient to finish the exercises and then rest. F. Assess the patient for additional symptoms of intolerance.

a, c, f. When a patient reports fatigue during range-of-motion exercises, the nurse should stop the activity, reevaluate the plan of care, and assess the patient for further symptoms indicating the activity is not tolerated. The exercises can be rescheduled for times of the day when the patient is feeling more rested, or spaced out at different times of the day.

The nurse uses blended competencies when caring for patients in a rehabilitation facility. Which interventions reflect the use of cognitive skills? Select all that apply. A. Monitoring for side effects of medications B. Safely administering an injectable medication C. Teaching a patient about diabetes and its management D. Acting as witness by signing a surgical consent form E. Helping a patient identify their progress in physical therapy F. Comforting a patient who has received bad news

a, c. Using critical thinking to teach a patient about a disease process and management and monitoring for side effects of medications are cognitive competencies. Performing an injection correctly is a technical skill; witnessing/signing an informed consent form is a legal/ethical action, and comforting a patient is an interpersonal skill.

During a nursing staff meeting to discuss delayed documentation, the nurses unanimously agree that they will ensure all vital signs are reported and charted within 15 minutes following assessments. This decision is consistent with which characteristics of effective communication? Select all that apply. A. Group decision making B. Group leadership C. Group power D. Group identity E. Group patterns of interaction F. Group cohesiveness

a, d, e, f. Solving problems involves group decision making; ascertaining the task is important and agreeing to complete the task on time is characteristic of group identity. Group patterns of interaction involve honest communication and member support; cohesiveness occurs when members generally trust each other, have a high commitment to the group, and a high degree of cooperation. Group leadership occurs when groups use effective styles of leadership to meet goals; with group power, sources of power are recognized and appropriately used to accomplish group outcomes.

A nurse admitting a patient to a long-term care facility performs a functional assessment. Which questions will the nurse include as part of the assessment? Select all that apply. A. "Are you able to dress yourself?" B. "Do you have a history of smoking?" C. "What is the problem for which you are seeking care?" D. "Do you prepare your own meals?" E. "Do you manage your own finances?" F. "Whom do you rely on for support?"

a, d, e. A functional health assessment including strengths and areas needing improvement focuses on the effects of health or illness on a patient's quality of life. Questions about the patient's ability to perform ADLs and IADLs such as dressing, grooming, preparing meals, and managing finances are included. A history of smoking is a lifestyle factor, and the chief complaint is the reason for seeking health care; both are assessed during the health history. Social networks and support psychosocial factors are also assessed during the health history.

A nurse is teaching a nursing student how to perform perineal care for patients. What actions are appropriate when performing this procedure? Select all that apply. A. For male and female patients, wash the groin area with a small amount of soap and water and rinse. B. For a female patient, spread the labia and move the washcloth from the anal area toward the pubic area. C. For male and female patients, always proceed from the most contaminated area to the least contaminated area. D. For male and female patients, use a clean portion of the washcloth for each stroke. E. For a male patient, clean the tip of the penis first, moving the washcloth in a circular motion from the meatus outward. F. In an uncircumcised male patient, avoid retracting the foreskin (prepuce) while washing the penis.

a, d, e. Wash and rinse the groin area (both male and female patients) with a small amount of soap and water, and rinse. For male and female patients, always proceed from the least contaminated area to the most contaminated area and use a clean portion of the washcloth for each stroke. For a male patient, clean the tip of the penis first, moving the washcloth in a circular motion from the meatus outward. For a female patient, spread the labia and move the washcloth from the pubic area toward the anal area. In an uncircumcised male patient (teenage or older), retract the foreskin (prepuce) while washing the penis and return it to its original position when finished.

A nurse is caring for a group of patients. Which actions are appropriate to include in the implementation phase of care? Select all that apply. A. Changing the dressings on a burn victim's arm B. Assessing a patient's nutritional intake C. Formulating a nursing diagnosis for a patient with epilepsy D. Turning a patient in bed every 2 hours to prevent pressure injuries E. Checking a patient's insurance coverage at the initial interview F. Determining availability of community resources for a patient with dementia

a, d, f. During the implementing step of the nursing process, nursing actions that were formulated during the planning process are carried out. The purpose of the implementation phase is to assist the patient in achieving valued health outcomes, for example promote health, prevent disease and illness, restore health, and facilitate coping with altered functioning. Assessing a patient's nutritional status or insurance coverage occurs in the assessment step, and formulating nursing diagnoses occurs in the diagnosing/analyzing step.

When performing a dressing change requiring surgical asepsis, a nurse opens sterile supplies and dons sterile gloves. What additional action by the nurse is appropriate? A. Avoiding splashing while pouring irrigant onto the sterile field B. Covering the nose and mouth with gloved hands if a sneeze is imminent C. Using forceps soaked in a disinfectant to place dressings on the sterile field D. Considering the outer 1 inch of the sterile field sterile

a, d. Considering the outer inch of a sterile field as contaminated is a principle of surgical asepsis. Moisture, such as from splashes contaminates the sterile field, and sneezing would contaminate the sterile gloves. Forceps soaked in disinfectant are not considered sterile.

At the last staff meeting, the nurse manager discussed the organizational initiative to improve provision of culturally competent care. During rounds, which behaviors inconsistent with this goal require the manager to intervene? Select all that apply. A. A staff nurse tells the AP that patients should not be given a choice, but should shower or bathe daily. B. A nurse asks the family of a patient who has died if they would like to wash their loved one's body. C. A nurse tells another nurse that Jewish dietary restrictions are just a way for them to get special foods. D. A Catholic nurse insists that a patient diagnosed with terminal bladder cancer see the chaplain in residence. E. A nurse obtains a translator to speak to the patient in their native language. F. A nurse refuses to care for a married gay patient who is HIV positive because the nurse is against same-sex marriage.

a, d. Cultural imposition occurs when someone believes others should conform to their beliefs, such as whether or not to shower or bathe daily, when a Catholic nurse insists that a terminally ill patient see a chaplain. Cultural blindness occurs when a nurse treats all patients the same regardless of culture. Culture conflict occurs when a nurse judges a patient's dietary restrictions as a way to get their favorite foods. When a nurse refuses to respect an older adult's ability to speak for themselves, or refuses to treat a patient based on their sexual orientation, stereotyping has occurred.

Nursing students approaching graduation and licensure are required to read the state nurse practice act. Which topics in the law will they identity as guides to professional practice? Select all that apply. A. Actions resulting in discipline B. Clinical procedures C. Medication administration D. Scope of practice E. Delegation policies F. Medicare reimbursement

a, d. Each state has a nurse practice act that protects the public by broadly defining the legal scope of nursing practice. Practicing beyond those limits makes nurses vulnerable to charges of violating the state nurse practice act. Nurse practice acts also list the violations that can result in disciplinary actions against nurses. Clinical procedures are covered by the health care institutions themselves. Medication administration and delegation are topics covered by the board of nursing. Laws governing Medicare reimbursement are enacted through federal legislation.

A nurse is developing an exercise program for a patient who has COPD. Which instructions would the nurse include in a teaching plan for this patient? Select all that apply. A. Teach the patient to avoid sudden position changes that may cause dizziness. B. Recommend that the patient restrict fluid intake until after exercise. C. Instruct the patient to push a little further beyond fatigue each session. D. Tell the patient to avoid exercising in very cold or very hot temperatures. E. Encourage the patient to modify exercise if weak or ill. F. Recommend that the patient consume a high-carb, low-protein diet.

a, d. Teaching points for exercising for a patient with COPD include avoiding sudden position changes that may cause dizziness and avoiding extreme temperatures. The nurse should also instruct the patient to remain adequately hydrated, respect fatigue as a sign of activity intolerance and not push to the point of exhaustion, and avoid exercise if weak or ill. Older adults should consume a high-protein, high-calcium, and vitamin D-enriched diet.

A nurse performs an assessment of a family consisting of a single parent, a grandparent, and two children. What interview questions will the nurse direct toward the mother to best determine the family's affective and coping functions? Select all that apply. A. Who is the person you depend on for emotional support? B. Who is the person you depend on for financial support in your family? C. Do you plan on having any more children? D. Who keeps your family together in times of stress? E. What family traditions do you pass on to your children?

a, d. The five major areas of family function are physical, economic, reproductive, affective and coping, and socialization. Affective areas of function include feelings and coping, assessed by determining who provides emotional support in times of stress. Assessing the financially responsible individual focuses on the economic function. Inquiring about having more children assesses the reproductive function, asking about family traditions assesses the socialization function, and checking the environment assesses the physical function.

What statements will the nurse use to explain the primary purpose of the nursing assessment? Select all that apply. A. "The nursing assessment will allow us to plan and deliver individualized, holistic nursing care that draws on your strengths." B. "It's hospital policy. I know we ask a lot of questions, but I will try to make this quick." C. "As a nursing student, I need to develop assessment skills about your health status and need for nursing care." D. "This validates that your responses with the medical exam are consistent and that all our data are accurate." E. "I will check your health status and see what kind of nursing care you may need." F. "This is to determine the necessity for referring your nursing care needs to a health care provider."

a, e, f. Medical assessments target data pointing to pathologic conditions, whereas nursing assessments focus on the patient's responses to actual and potential health problems. The initial comprehensive nursing assessment results in baseline data that enable the nurse to make a judgment about a patient's health status, the ability to manage their own health care and the need for nursing. It also helps nurses plan and deliver individualized, holistic nursing care that draws on the patient's strengths and promotes optimum functioning, independence, and well-being, and enables the nurse to refer the finding(s) to the health care provider or collaborate with other health care professionals where indicated. Citing hospital policy or student learning is a secondary reason, and although it may be true that a nurse may need to develop assessment skills, it is not the main reason for a nursing history and assessment. The assessment augments the medical examination but is not performed to check its accuracy.

A nurse is caring for an adolescent with severe acne. Which recommendations would be most appropriate to include in the teaching plan for this patient? Select all that apply. A. Wash the skin twice a day with a mild cleanser and warm water. B. Use cosmetics liberally to cover blackheads. C. Apply emollients on the area. D. Squeeze blackheads as they appear. E. Keep hair off the face and wash hair daily. F. Avoid tanning booth exposure and use sunscreen.

a, e, f. Washing the skin removes oil and debris, hair should be kept off the face and washed daily to keep oil from the hair off the face. Exposure to UV light should be avoided, especially when using acne treatments. Liberal use of cosmetics and emollients can clog the pores, worsening acne. Squeezing blackheads is discouraged because it may lead to infection.

A registered nurse acts as nurse coach to provide teaching to patients who are recovering from a stroke. Which statement directs the nurse in performing this role? A. The nurse uses discovery to identify the patients' personal goals and create a plan that will result in change. B. The nurse is the expert in providing teaching and education strategies to provide dietary and activity modifications. C. The nurse becomes a mentor to the patients and encourages them to create their own fitness programs. D. The nurse assumes an authoritative role to design the structure of the coaching session and support the achievement of patient goals.

a. A nurse coach establishes a partnership with a patient and, using discovery, facilitates the identification of the patient's personal goals and agenda to lead to change rather than using teaching and education strategies with the nurse as the expert. A nurse coach explores the patient's readiness for coaching, designs the structure of a coaching session, supports the achievement of the patient's desired goals, and with the patient determines how to evaluate the attainment of patient goals.

A nurse notices a patient is walking to the bathroom with a stooped gait, facial grimacing, and grunting sounds. Based on these nonverbal cues, what action will the nurse take next? A. Assess for pain and the need for analgesia. B. Ask the patient if they feel anxious. C. Offer to sit with the patient and listen to their feelings. D. Suggest the patient increase their fluid intake to prevent constipation.

a. A patient who presents with nonverbal communication of a stooped gait, facial grimacing, and grunting sounds is most likely communicating pain. The nurse should clarify this nonverbal behavior.

A nursing student obtains a blood pressure reading of 148/100. To determine the significance of this reading, what action will the nurse take first? A. Comparing this reading to standards and trends in the medical record B. Checking the taxonomy of nursing diagnoses for a pertinent label C. Checking a medical text for the signs and symptoms of high blood pressure D. Consulting with experienced nurse colleagues

a. A standard, or a norm, is a generally accepted rule, measure, pattern, or model to which data can be compared within the same class or category. When interpreting the significance of a patient's blood pressure reading, the nurse uses normative values for the patient's age group, race, and illness category and compares these to the patient's recent results. Identifying the reason for deviation from a norm gives direction to the etiology of a health problem (e.g., insufficient knowledge, nutrition, stress, and coping, or other).

A nursing student is on clinical rotation in a long-term care facility. Which action best reflects the student acting as advocate? A. Telling the health care team that a patient clearly stated they do not want to be transported to the hospital B. Avoiding input in care conferences, as patient advocacy is primarily performed by the health care provider C. Assisting the primary nurse in making good health care decisions for patients and residents D. Deferring to whatever decisions patients and residents want

a. Advocacy is the protection and support of another's rights. Among the patients with special advocacy needs are the very young and the very old, those who are seriously ill, and those with disabilities. Patient advocacy is the responsibility of every member of the professional caregiving team—not just nurses. Nurse advocates do not make health care decisions for their patients and residents; rather, they facilitate patient decision making. Advocacy does not entail supporting patients in all their preferences.

An oncology nurse is analyzing a patient's strengths and finds the patient is well educated, learns quickly, and is resilient. In which phase of the nursing process will the nurse use this information? A. Diagnosing B. Evaluating C. Planning D. Implementing

a. Assessing for strengths and weaknesses is the first step of the nursing process, which has been completed. Next, the nurse clusters cues and develops diagnoses that give rise to interventions. Evaluating the plan is followed by completing or modifying the plan.

A nurse tells a patient, "tonight's menu selection is pork. I understand many people in your culture do not eat pork; may I order something else for you?" When the patient states they no longer observe this dietary practice, the nurse understands that the patient has experienced what transition? A. Cultural assimilation B. Cultural imposition C. Culture shock D. Ethnocentrism

a. Assimilation occurs when minority groups living within a dominant group lose the cultural characteristics that make them different. Cultural imposition occurs when one person believes that everyone should conform to their own belief system. Culture shock occurs when a person is placed in a different culture perceived as strange, and ethnocentrism is the belief that the ideas, beliefs, and practices of one's own cultural group are best, superior, or most preferred to those of other groups.

A nursing unit has adopted use of a care bundle for insertion of central venous catheters. During the procedure, which action by a nurse requires the charge nurse to intervene? A. They discard the sterile drapes in the insertion kit. B. The primary nurse reminds everyone in the room to wear a mask. C. The team includes every item in the bundle during the procedure. D. The nursing student states using the bundle improves patient outcomes.

a. Care bundles are sets of evidence-based interventions that, when performed together and consistently, improve the process of care and patient outcomes. Discarding sterile drapes from the insertion kit circumvents this process.

A group of students in a community health course is discussing disaster management. Which nursing leader will the students identify as the founder of the Red Cross? A. Clara Barton B. Florence Nightingale C. Lillian Wald D. Lavinia Dock

a. Clara Barton established the Red Cross in the United States in 1882. Florence Nightingale elevated the status of nursing to a respected occupation, improved the quality of nursing care, and founded modern nursing education. Lillian Wald was the founder of public health nursing. Lavinia Dock was a nursing leader and women's rights activist instrumental in establishing women's right to vote.

The nursing assessment of a patient with a diagnosis of anorexia nervosa reveals the patient consumes a vegan diet of 700 calories daily and has lost 30 lb in 4 months. The nurse's recommendation to meet with a nutritionist is the outcome of which process? A. Clinical judgment B. Nursing process C. Clinical reasoning D. Critical thinking

a. Clinical judgment is the outcome of critical thinking and clinical reasoning, using the nursing process as a framework. Clinical reasoning refers to ways of thinking about patient care issues including weighing and validating options (determining, preventing, and managing patient problems). Critical thinking includes reasoning both outside and inside of the clinical setting.

A nurse writes the outcome for a patient who is trying to lose weight: "The patient will explain the relationship between weight loss, increased exercise, and decreased calorie intake." This outcome reflects which domain of learning? A. Cognitive B. Psychomotor C. Affective D. Physical changes

a. Cognitive outcomes involve increases in patient knowledge; psychomotor outcomes describe the patient's achievement of new skills; affective outcomes pertain to changes in patient values, beliefs, and attitudes; and physical changes are actual bodily changes in the patient (e.g., weight loss, increased muscle tone).

A nurse is practicing community-based nursing in a mobile health clinic. Which example best demonstrates community-based nursing? A. Caring for a mother and her child who have diabetes B. Providing shelter for vulnerable populations within the community C. Providing local same-day surgery facilities D. Assisting families in crisis and overseeing the crisis hotline

a. Community health nursing focuses on whole populations within a community, and community-based nursing is centered on the health care needs of individuals and families. Nurses practicing community-based nursing provide interventions to manage acute or chronic health problems, promote health, and facilitate self-care.

A nurse is preparing a teaching plan for a patient with asthma on the use of an inhaler. What teaching method is most appropriate for this patient? A. Demonstration B. Lecture C. Discovery D. Panel session

a. Demonstration of techniques, procedures, exercises, and the use of special equipment is an effective patient-teaching strategy for a skill. Lecture can be used to deliver information to a large group of patients but is more effective when the session is interactive; it is rarely used for individual instruction, except in combination with other strategies. Discovery is a good method for teaching problem-solving techniques and independent thinking. Panel discussions can be used to impart factual material but are also effective for sharing experiences and emotions.

A nurse historian is researching changes in professional nursing and nursing education in the mid-20th century. What trend promoted advancement of the profession and of nursing education? A. Increased need for nurses and knowledge explosion led to emphasis on education. B. Focus on the war effort slowed development of knowledge in medicine and technology. C. Role of the nurse focused on acute technical skills used in hospital settings. D. Nursing care was dependent on the medical profession to define its priorities

a. During World War II, large numbers of women worked outside the home. They became more independent and assertive, leading to an increased emphasis on education. The war itself created a need for more nurses and resulted in a knowledge explosion in medicine and technology.

A nurse is performing a breast assessment using the circular technique for palpation, gently compressing the breast tissue against the chest wall. How does the nurse proceed with the examination? A. Begins at the tail of Spence and moves in increasing smaller circles B. Starts at the outer edge of the breast and palpates up and down the breast C. Works in a counterclockwise direction and palpates from the periphery toward the areola D. Proceeds from the inner edge of the breast and palpates up and down the breast

a. During breast assessment, the nurse palpates each quadrant of the breasts in a systematic method using the pads of the first three fingers to gently compress the breast tissue against the chest wall. In the circular method, the nurse begins at the tail of Spence and move in increasingly smaller circles. In the wedge method, the nurse works in a clockwise direction and palpates from the periphery toward the areola. In the vertical strip method, the nurse begins at the outer edge of the breast, palpating up and down the breast.

A nurse is administering enoxaparin subcutaneously using the manufacturer's prefilled syringe. Which action reflects correct practice? A. Administering the medication in the posterolateral abdomen B. Removing air bubble prior to injecting C. Gently rubbing the abdomen after injection to promote absorption D. Replacing the cap on the syringe before discarding it in the sharps container

a. Enoxaparin should be administered in the alternating "left and right anterolateral and left and right posterolateral abdominal wall[s]." Air, inserted by the manufacturer, should not be expelled prior to use. As this is a low-molecular-weight heparin, do not massage the area to prevent bruising. Recapping any syringe places the nurse at risk for needlestick injury and is not recommended.

A nurse is using general systems theory to assist a family of four develop healthier food choices. What statement best reflects a key point of this theory? A. Food choices made by the parents will influence choices made by the children. B. The children should avoid giving feedback on the family food choices. C. Boundaries are closed between family members and their environments. D. A change in one family member's behavior rarely affects other members.

a. General systems theory defines a system as a set of interacting elements contributing to the overall goal of the system. A change in one element could affect other subsystems, as well as the whole. To survive, open systems maintain balance through feedback, in this case, family members. An open system allows energy, matter, and information to move freely between systems and boundaries, such as with members of the health care team, whereas a closed system does not allow input from or output.

When administering an IVPB medication using gravity, what action is appropriate for the nurse take? A. Placing the primary IV bag below the level of the piggyback bag B. Disconnecting the tubing closest to the patient and flushing the intravenous access C. Ensuring the piggyback bag is below the main IV bag D. Closing the roller clamp to the secondary infusion

a. Hang the piggyback container on the IV pole, positioning it higher than the primary IV bag. When infusing by gravity, the primary IV fluid container must be lowered. Closing the roller clamp to the secondary line will impede flow of the piggybacked medication.

A nurse is using the PIE format to document care of a patient who is diagnosed with type 2 diabetes. What information does the nurse need to complete documentation in this format? A. Patient problem list B. Narrative notes describing the patient's condition C. Overall trends in patient status D. Planned interventions and patient outcomes

a. In the PIE format, patient problems are numbered; documented in the progress notes; worked up using the Problem, Intervention, Evaluation (PIE) format; and evaluated each shift. Resolved problems are dropped following the nurse's review. Narrative notes allow nurses to describe a condition, situation, or response in their own terms. Overall trends in patient status can be seen immediately when using charting by exception (CBE). Planned interventions and patient-expected outcomes are the focus of the case management model.

When inspecting the skin of a patient who has cirrhosis of the liver, the nurse notes the skin has a yellow tint. What term will the nurse use to document the skin assessment in the electronic health record? A. Jaundice B. Cyanosis C. Erythema D. Pallor

a. Jaundice refers to a yellowish skin color caused by liver, gallbladder, or pancreatic exocrine diseases. Cyanosis is a bluish skin color caused by a cold environment or decreased oxygenation. Erythema is a reddish color caused by blushing, alcohol intake, fever, injury trauma, or infection. Pallor is a paleness caused by anemia or shock

A nurse notices that an older adult patient is malnourished, and blood tests reveal reduced plasma protein levels. For which of these pharmacodynamic effects will the nurse observe? A. Medication toxicities B. Failure to thrive C. Weight gain D. Mental confusion

a. Lowered protein levels in the body means less drug bound to plasma proteins, which leads to a higher concentration of free (unbound) drug in the body. This higher drug concentration increases the risk for adverse effects.

A charge nurse meets with staff to outline a plan to provide transcultural nursing care for patients in their health care facility. The charge nurse explains that transcultural care as the central theme of nursing care, knowledge, and practice was promoted by which theorist? A. Madeline Leininger B. Jean Watson C. Dorothy E. Johnson D. Betty Newman

a. Madeline Leininger's theory provides the foundations of transcultural nursing care by making caring the central theme of nursing. Jean Watson stated that nursing is concerned with promoting and restoring health, preventing illness, and caring for the sick. The central theme of Dorothy E. Johnson's theory is that problems arise because of disturbances in the system or subsystem or functioning below optimal level. Betty Newman proposed that humans are in constant relationship with stressors in the environment and the major nursing focus is keeping the patient system stable through accurate assessment of these stressors.

A nurse is preparing to administer medications to a patient transferred from the intensive care unit just as lunch is served. Prior to administering medications to the patient, the nurse takes which action? A. Performing medication reconciliation B. Morning care has been administered C. Ordering the patient a meal D. Taking a report from the nurse sending the patient

a. Medication reconciliation has been shown to decrease the incidence of medication errors that occur at points of transition in patient care. This process maintains a current, accurate list of medications a patient has received. Morning care can be delayed, if not yet given. Obtaining the patient's meal can be delegated to another person on the unit (e.g., AP). A report must be taken prior to accepting the patient for transfer.

A public health nurse is planning interventions for a community and plans to determine the frequency of diseases in the area. What information will best guide the nurse? A. Morbidity table B. Disease eradication statistics C. Mortality rates D. Annual hospital admissions

a. Morbidity refers to how frequently a disease occurs; mortality is the number of deaths resulting from a disease. These statistics may be placed in tables as a reference for others.

A nurse is engaged in community health promotion activities. What activity best reflects this type of health promotion? A. Teaching about fall prevention and home safety at a senior center B. Holding a blood glucose screening for diabetes at a local church C. Educating hospice volunteers on therapeutic communication D. Providing education on crutch walking to a child with a fractured tibia

a. Nurses promote health and prevent illness primarily by teaching, through personal example and administering preventative vaccinations. Performing blood glucose screening for diabetes detects a disease after it has developed, rather than diabetes prevention. Therapeutic communication during hospice care and crutch walking are additional interventions used for diseases or health problems already present.

A nursing student tells the primary nurse that their patient has not had a bowel movement for 2 days and suggests adding the health problem "Constipation" to the care plan. How would the nurse best respond? A. "Did you assess the patient's usual bowel patterns and appearance of the last stool?" B. "This early diagnosis will help us manage the problem before it becomes severe." C. "Have you determined if this is an actual or a possible diagnosis?" D. "This condition requires a medical diagnosis."

a. Patient health problems are derived from clusters of related data and patterns, rather than a single cue. The nurse determines if this is the patient's usual bowel pattern, or whether an underlying reason exists for the lack of a bowel movement. Constipation is a health problem the nurse can resolve with independent or interdependent nursing actions.

A nurse is caring for a patient with type 2 diabetes who has an infected foot ulcer requiring dressing changes. Which nursing action best demonstrates the QSEN competency of patient-centered care? A. Asking the patient if they would like their spouse to be present for a teaching session B. Researching new procedures to care for foot ulcers when developing a care plan for this patient C. Leading a grand rounds or unit-based discussion on complications of diabetes D. Using the electronic medical record to review trends of the patient's blood glucose levels

a. Patient-centered care commits to developing caring relationships based on mutual trust to communicate and deliver care based on patient preferences and values. Evidence-based practice integrates the best current evidence for safe practice with clinical expertise. Teamwork and collaboration shares patient information or opportunities for learning with others. Informatics manages patient information, mitigates error, and supports decision making using the electronic medical record and other databases.

A school nurse determines that a student who has lost weight is at risk for an eating disorder and would benefit from a nutritional assessment. What action will the nurse take? A. Perform a focused nutritional assessment B. Seek direction from the student's health care provider C. Suggest the student visit the nurse-run clinic D. Request a consultation with a nutritionist

a. Performing a focused assessment is an independent nurse-initiated intervention, not requiring a prescription from or intervention by the health care provider, advanced practice nurse, or nutritionist.

A nurse working in a long-term care facility uses proper principles of ergonomics when moving and transferring patients to avoid back injury. Which action should be the focus of these preventive measures? A. Carefully assessing the patient care environment B. Using two nurses to lift a patient who cannot assist C. Wearing a back belt to perform routine duties D. Properly documenting the patient lift

a. Preventive measures should focus on careful assessment of the patient care environment so that patients can be moved safely and effectively. Using lifting teams and assistive patient handling equipment rather than two nurses to lift increases safety. The use of a back belt does not prevent back injury. The methods used for safe patient handling and mobility should be documented but are not the primary focus of interventions related to injury prevention.

A nurse is caring for a patient with exacerbation of COPD and pneumonia. When auscultating the lungs, coarse expiratory, low-pitched, and continuous sounds that clear with coughing are present. How will the nurse document this breath sound in the electronic health record? A. Rhonchi B. Bronchovesicular breath sounds C. Stridor D. Bronchial breath sounds

a. Rhonchi are abnormal low-pitched, continuous breath sounds auscultated during inspiration and occasionally expiration, indicating that air is passing through or around secretions. Bronchovesicular breath sounds are normal sounds heard on inspiration and expiration. Stridor is a harsh, loud, high-pitched sound auscultated on inspiration indicating narrowing of the upper airway or presence of a foreign body. Bronchial sounds are normal blowing, hollow sounds, auscultated over the larynx and trachea.

A nurse works in a long-term care facility where standing orders are in place for influenza vaccines for all residents. What is the nurse's priority, when carrying out the prescriptions? A. Assessing whether the patient previously received the vaccine B. Refusing to give the vaccine without a written prescription C. Determining if the standing orders are inappropriate for their unit D. Calling the nursing supervisor to determine if this is a permitted action

a. Standard orders empower the nurse to initiate actions ordinarily requiring the order, prescription or supervision of a health care provider. The nurse first assesses whether the patient has already received the vaccine. The standing order is a valid prescription given to cover common, recurring actions the nurse can use when indicated.

Nurses in the United States follow regulations outlined by each state's nurse practice act. A nurse moving to a different state anticipates finding which elements are common to all states' nurse practice acts? A. Definition of the legal scope of nursing practice for those registered B. Provision of continuing education programs to meet state requirements C. Determining the scope of content covered in the NCLEX examination D. Creating institutional policies for health care practices

a. State nurse practice acts regulate practice by defining the legal scope of practice, creating a state board of nursing to make and enforce rules and regulations, define important terms and activities in nursing, and establish criteria for the education and licensure of nurses. The state's board of nursing has the legal authority define the education required to take NCLEX but does not provide that education or determine the content of the examination. The board has the legal authority to allow graduates of approved schools to take the licensing examination. Institutional policies are created by the institutions themselves.

The charge nurse tells a nursing student to change a surgical dressing while they take care of other patients. The student has not changed dressings before and does not feel confident performing the procedure. What action should the student take? A. Tell the charge nurse that they lack the technical competencies to change the dressing independently B. Assemble the equipment for the procedure and follow the steps in the procedure manual C. Ask another student nurse to work collaboratively with them to change the dressing D. Tell the clinical instructor they have not had experience with the delegated task

a. Student nurses should notify their nursing instructor or nurse preceptor if they believe they lack any competencies needed to safely implement the care plan. Once educated and technically prepared, the nursing student may perform a dressing change.

The nurse preceptor and a new graduate nurse on the surgical unit are performing preoperative assessments on a group of patients. What statement by the graduate nurse requires the preceptor to intervene? A. "I am sure everything will be fine; you have nothing to worry about." B. "When you return from surgery, you'll need to cough and deep breathe." C. "Many people on this unit have had that procedure with good success." D. "You seem fearful, can I answer any questions about the procedure?"

a. Telling a patient that everything will be fine is a cliché. This statement gives false assurance and may give the patient the impression that the nurse is dismissive of a patient's concerns or condition.

Thirty-six hours after having surgery, a patient has a slightly elevated body temperature and generalized malaise as well as pain with redness at the surgical site. Which action is most appropriate? MAR -Acetaminophen 650 mg every 6 hours prn fever -Cefazolin (antibiotic) 1 g 1 hour preoperatively -Cefazolin 1 g, every 6 hours 3 times, postoperatively A. Documenting the findings and continuing to monitor the patient B. Administering antipyretics and contacting the provider for an antibiotic prescription C. Increasing the frequency of assessment to every hour and notifying the patient's primary care provider D. Obtaining a wound culture and increasing the frequency of wound care

a. The assessment findings are normal for this stage of healing following surgery. The patient is in the inflammatory phase of the healing process, which involves a response by the immune system. This acute inflammation is characterized by pain, heat, redness, and swelling at the site of the injury (surgery, in this case). The patient also has a generalized body response, including a mildly elevated temperature, leukocytosis, and generalized malaise.

A nursing student tells the clinical instructor that their patient is fine and has "no complaints." Which question by the faculty coaches the student to provide evidence that supports their assessments? A. "Could you tell me how you validated this?" B. "Do you think your patient feels free to share their concerns?" C. "That's good to hear. Tell me about the care you provided." D. "Please reassess the patient; they were admitted with a serious problem."

a. The instructor is reminding the student that all data must be validated. Questioning the use of the word "fine" allows the nurse to determine if this is a social and reflexive response, and there may be another need the nurse can meet. Concluding that the patient does not trust the student is premature and is based on an invalidated inference. Saying "That's good to hear" and asking the student to describe the care provided is incorrect because it accepts the invalidated inference. Telling the student to reassess the patient because they were admitted with a serious problem is incorrect because it is possible that the condition is resolving.

A nurse tells a new mother from Africa that she should not carry her infant in a sling because bassinets are safer. The charge nurse suggests the nurse is displaying which behavior? A. Cultural imposition B. Clustering C. Cultural competency D. Stereotyping

a. The nurse is trying to impose her belief that bassinets are preferable to baby slings on the African mother—in spite of the fact that African women have safely carried babies in these slings for years.

A public health nurse is leaving the home of a young mother who has an infant with special needs. The neighbor states, "How is she doing, since the baby's father is no help?" What is the nurse's best response? A. "New mothers need support." B. "The lack of a father is difficult." C. "How are you today?" D. "It is a very sad situation."

a. The nurse must maintain confidentiality when providing care. The statement "New mothers need support" is a general statement that all new parents need help. The statement is not judgmental of the family's roles. "How are you today?" is dismissive of the neighbor's question.

A nurse is administering a pain medication to a patient. In addition to checking the identification bracelet, which active identification strategy reflects best practice? A. Asking the patient their name and birthdate B. Reading the patient's name on the sign over the bed C. Asking the patient's roommate to verify the patient's name D. Asking, "Are you Mr. Brown?"

a. The nurse should ask the patient to state their name and birthdate based on facility policy. A sign over the patient's bed may not be current or correct. The roommate is an unsafe source of information. The patient may not hear their name or may reply in the affirmative regardless of accuracy (e.g., a person with a hearing deficit).

An attorney representing a patient's family who is suing for wrongful death calls the nurse to obtain a better understanding of the nurse's actions. How will the nurse respond? A. "I can't talk with you; you will have to contact my attorney." B. "I will answer your questions, so you'll understand how the situation occurred. C. "I hope I won't be blamed for the death because it was so busy that day." D. "First tell me why you are doing this to me. This could ruin my career!"

a. The nurse should not discuss the case with anyone at the facility (except the risk manager), with the plaintiff, with the plaintiff's lawyer, with anyone testifying for the plaintiff, or with reporters. This is one of the cardinal rules for nurse defendants.

A nurse is instructing a patient recovering from a stroke on proper use of a cane. What information will the nurse include in the teaching plan? A. Support weight on the stronger leg and cane and advance weaker foot forward. B. Hold the cane in the same hand of the leg with the most severe deficit. C. Stand with as much weight distributed on the cane as possible. D. Avoid using the cane to rise from a sitting position, as this is unsafe.

a. The proper procedure for using a cane is to (1) stand with weight distributed evenly between the feet and cane; (2) support weight on the stronger leg and the cane and advance the weaker foot forward, parallel with the cane; (3) support weight on the weaker leg and cane and advance the stronger leg forward ahead of the cane; (4) move the weaker leg forward until even with the stronger leg and advance the cane again as in step 2. The patient should keep the cane within easy reach and use it for support to rise safely from a sitting position.

A primary nurse is preparing a discharge plan for a patient who has been hospitalized following a double mastectomy. Which statement is most appropriate for the nurse to use in the termination phase of the therapeutic relationship? A. "Let's review the progress you've made in meeting your goals." B. "I'd like to review your medication schedule with you." C. "I need to document today's teaching session in the electronic health record." D. "Should we include your family in today's session?"

a. The termination phase occurs when the conclusion of the initial agreement is acknowledged. Discharge planning correlates with the termination phase of a therapeutic relationship and the progress toward the patient's goals are reviewed.

A nurse is caring for a patient with lower extremity paralysis. Which action will the nurse take to prevent external rotation of the hip and foot? A. Use a trochanter roll. B. Apply SCDs. C. Obtain a prescription for antiembolism stockings. D. Have the patient maintain low-Fowler's position. E. Have the patient cross their arms on their chest and place a pillow between their knees. F. Place a cervical collar on the patient's neck and gently roll them to the other side of the bed.

a. The trochanter roll is used to support the hips and legs to prevent external rotation. SCDs and antiembolism stockings are used to prevent DVT. Fowler's position allows for foot rotation and increases sacral pressure.

A nurse caring for a patient with a stage 3 pressure wound with tunneling. How will the nurse best assess the tunneled area? A. Moisten a sterile, flexible applicator with saline and insert it gently into the wound at a 90-degree angle with the tip down. B. Photograph the wound per policy and describe the estimated depth in centimeters. C. Gently insert a sterile applicator into the wound and move it in a clockwise direction. D. Insert a calibrated probe gently into the wound and mark the point that is even with the surrounding skin surface.

a. To measure the depth of a wound, the nurse should perform hand hygiene and apply gloves; moisten a sterile, flexible applicator with saline and insert it gently into the wound at a 90-degree angle with the tip down; mark the point on the swab that is even with the surrounding skin surface, or grasp the applicator with the thumb and forefinger at the point corresponding to the wound's margin; and remove the swab and measure the depth with a ruler.

A staff nurse tells a new graduate nurse not to bother studying too hard, since most clinical reasoning becomes second nature and intuitive once they begin practicing. Which response by the student is appropriate? A. Intuitive problem solving comes with years of practice and observation based on nursing knowledge and science. B. For nursing to remain a science, nurses must continue to be vigilant about avoiding intuitive reasoning. C. The emphasis on logical, scientific, evidence-based reasoning has held nursing back; we need intuitive, creative thinkers. D. The nurse's preference dictates whether they are logical, scientific thinkers or intuitive, creative thinkers.

a. When intuition is used alone, increased risks and fewer benefits may occur. Beginning nurses must use nursing knowledge and scientific problem solving as the basis of care; intuitive problem solving comes with years of practice and observation. If the beginning nurse has an intuition about a patient, that information should be discussed with the faculty member, preceptor, or supervisor. There is a place for intuitive reasoning in nursing, but it will augment, not replace logical, scientific reasoning. Critical thinking is contextual and changes depending on the circumstances, not on personal preference.

The nurse is caring for a patient 1 day postoperative abdominal surgery. The nurse identifies the patient is at risk for wound dehiscence. What patient risk factor is consistent with development of this problem? A. Cigar smoker B. Wound drainage 120 mL over 24 hours C. Height, 5′ 6″ and weight 240 lb D. WBC count 9,500 c/mm3

a. Wound dehiscence is the partial or total separation of wound layers as a result of excessive stress on unhealed wounds. Patients at greater risk include obese or malnourished individuals; tobacco smokers; and those taking anticoagulants, who have infected wounds, or who experience excessive coughing, vomiting, or straining (Hinkle & Cheever, 2018). An increase in the flow of (serosanguineous) fluid from the wound between postoperative days 4 and 5 may be a sign of an impending dehiscence. The patient may say that "something has suddenly given way."

Nursing students are discussing the care-based approach to ethical practice. What actions will the students ensure are included in the discussion? Select all that apply. A. Understanding that the needs of the many prevail versus the needs of the few B. Promoting the dignity and respect of patients as people C. Attending to the individual attributes of each patient D. Cultivating responsiveness to others and professional responsibility E. Understanding that moral skills include kindness, attentiveness, compassion, and reliability

b, c, d, e. A care-based approach, essential to thoughtful, person-centered care, directs attention to the specific patient situation, viewed within the context of their life narrative. The care-based approach includes the following characteristics: centrality of the caring relationship; promotion of patient dignity and respect; attention to patient particulars; cultivation of responsiveness to others; professional responsibility; and redefinition of fundamental moral skills like kindness, attentiveness, empathy, compassion, and reliability. An action is right or wrong based on a rule, independent of its consequences, such as "the needs of the many outweigh the needs of the few."

A graduate nurse and preceptor are discussing protected health information (PHI) and HIPAA laws. The preceptor explains that PHI can be released without the patient's signed authorization in which situations? Select all that apply. A. News media are preparing to report on a patient who is a public figure. B. Data are needed for the tracking and notification of disease outbreaks. C. Protected health information is needed by a coroner. D. Child abuse and neglect are suspected. E. Protected health information is needed to facilitate organ donation. F. The sister of a patient with Alzheimer's disease wants to help provide care.

b, c, d, e. According to HIPAA, a health institution may share PHI without written patient authorization for tracking disease outbreaks, infection control, statistics related to dangerous problems with drugs or medical equipment, investigation and prosecution of a crime, identification of victims of crimes or disaster, reporting incidents of child abuse, neglect or domestic violence, responding to a valid subpoena, and providing information needed by coroners, medical examiners, funeral directors, and law enforcement in the case of a death from a potential crime and for facilitating organ donations. The nurse does not provide information to a news reporter without the patient's express authorization; typically, a hospital representative communicates with the media. A patient who has Alzheimer's disease will still need a release to share information; this may be given by the power of attorney.

A nurse is caring for a patient who presents with dyspnea, tachypnea, productive cough, fever, and low oxygen saturation. When developing the nursing care plan, which health problems might the nurse identify for this patient? Select all that apply. A. Bronchial pneumonia B. Impaired gas exchange C. Impaired Respiratory System Function D. Altered breathing pattern E. Impaired Thermoregulation

b, c, d, e. Nursing diagnoses are actual or potential health problems that can be prevented or resolved by independent or interdependent nursing interventions. These include Impaired gas exchange, supported by low oxygen saturation; Impaired Respiratory System Function; Altered breathing pattern, supported by dyspnea and tachypnea; and Impaired Thermoregulation. Pneumonia is a medical diagnosis.

Nurses from hospital in a low-income area of the city are developing educational materials and programs focused on health care for vulnerable populations. When planning community outreach, which members of the community will the nurses include? Select all that apply. A. White male diagnosed with HIV B. Black teenager who is 6 months pregnant C. Hispanic male who has type 2 diabetes D. Low-income family living in rural America E. Middle-class teacher living in a large city F. White infant born with cerebral palsy

b, c, d, f. National trends in the prevention of health disparities are focused on vulnerable populations, such as racial and ethnic minorities, those living in poverty, women, children, older adults, residents of rural areas and lower-income areas of cities, and people with disabilities and special health care needs

A nurse is developing a care plan for an older adult patient who is recovering from a hip arthroplasty (hip replacement). Which assessment findings indicate a high risk for this patient to develop area(s) of pressure injury? Select all that apply. A. The patient takes time to think about responses to questions. B. The patient is an older adult with a poor appetite. C. The patient reports inability to control their urine. D. The patient's albumin level is <3.2 mg/dL (normal, 3.4 to 5.4 g/dL). E. Lab findings include BUN 12 (older adult, normal 8 to 23 mg/dL) and creatinine 0.9 (adult female, normal 0.61 to 1 mg/dL). F. The patient reports increased pain in right hip when repositioning in bed or chair.

b, c, d, f. Pressure, friction, and shear, as well as other factors, usually combine to contribute to pressure injury development. The skin of older adults is more susceptible to injury; incontinence contributes to prolonged moisture on the skin, as well as negative effects related to urine in contact with skin; hip surgery involves decreased mobility during the postoperative period, as well as pain with movement, contributing to immobility; and increased pain in the hip may contribute to increased immobility. A low albumin level signals a risk for poor wound healing related to malnutrition. Apathy, confusion, and/or altered mental status are risk factors for pressure injury development; however, taking time to formulate responses is consistent with normal aging. This patient's BUN and creatinine are within normal range; however, dehydration (indicated by an elevated BUN and creatinine) is a risk for pressure injury development.

A charge nurse in a skilled nursing facility is working to reduce patients' foot and nail problems. The charge nurse reminds the nurses and APs to closely observe which of these patients at higher risk? Select all that apply. A. Patient taking antibiotics for chronic bronchitis B. Patient with type 2 diabetes C. Patient who has obesity D. Patient who frequently bites their nails E. Patient with prostate cancer F. Patient who frequently washes their hands

b, c, d, f. Variables known to cause nail and foot problems include deficient self-care abilities, vascular disease, arthritis, diabetes mellitus, history of biting nails or trimming them improperly, frequent or prolonged exposure to chemicals or water, trauma, ill-fitting shoes, and obesity. Antibiotic use and prostate cancer do not predispose to foot or nail problems.

A nurse caring for patients in the intensive care unit develops values from their experiences when forming a personal code of ethics. Which statements correctly guide this process? Select all that apply. A. People are born with values. B. Values act as standards to guide behavior. C. Values are ranked on a continuum of importance. D. Values influence beliefs about health and illness. E. Value systems are not related to personal codes of conduct. F. Nurses should not let their values influence patient care.

b, c, d. A value is a belief about the worth of something, about what matters, that acts as a standard to guide our behavior. A value system is an organization of values in which each is ranked along a continuum of importance, often leading to a personal code of conduct. A person's values influence beliefs about human needs, health, and illness; the practice of health behaviors; and human responses to illness. Values guide the practice of nursing care. An individual is not born with values; rather, values are formed during a lifetime from information from the environment, family, and culture.

The nurse on a medical-surgical unit is admitting a patient with a diagnosis of active tuberculosis. Which infection control precautions will the nurse put in place? A. Wearing sterile gloves for patients with visible body fluids B. Placing the patient on airborne precautions C. Wearing an N95 respirator mask when in the room D. Placing the patient in a single-occupancy room E. Ensuring the room provides positive pressure F. Restricting visitors for the duration of the patient's stay

b, c, d. Airborne precautions are used for patients who have infections with small particles that spread through the air, for example, tuberculosis, varicella, and rubeola. An N95 respirator mask is worn and the patient placed in a private room, preferably with negative air pressure. Sterile gloves are used for procedures requiring surgical asepsis. Standard precautions are for all patient care when contact with blood or body fluids, nonintact skin, and mucous membranes are likely. Visitors must wear PPE, including a mask.

A nurse is planning care for a patient admitted to the hospital for treatment of a drug overdose. What actions will the nurse take during the outcome identification and planning step of the nursing process? Select all that apply. A. Formulating nursing diagnoses B. Identifying expected patient outcomes C. Selecting evidence-based nursing interventions D. Explaining the nursing care plan to the patient E. Assessing the patient's mental status F. Evaluating the patient's outcome achievement

b, c, d. During the outcome identification and planning step of the nursing process, the nurse, patient, and family collaborate to establish priorities and identify and write expected patient outcomes. The nurse selects evidence-based nursing interventions, and communicates the care plan. These steps may overlap; however, formulating and validating nursing diagnoses are typically performed during the diagnosing step. Assessing mental status is part of the assessment step, and evaluating patient outcomes occurs during the evaluation step of the nursing process.

The nurse places a patient in the dorsal recumbent position during a physical assessment. Which nursing assessments can the nurse perform with the patient in this position? Select all that apply. A. Assessing the abdomen B. Taking peripheral pulses C. Performing a breast examination D. Auscultating the heart E. Assessing vital signs F. Assessing balance and gait

b, c, d. In the dorsal recumbent position, the patient lies on the back with legs separated, knees flexed, and soles of the feet on the bed. It is used to assess the head, neck, anterior thorax, lungs, heart, breasts, extremities, and peripheral pulses. It should not be used for abdominal assessment because it causes contraction of the abdominal muscles. Vital sign assessment should be done in the sitting position, and evaluating balance and gait is done with the patient in the standing position.

An RN working on a hospital unit frequently delegates patient care to assistive personnel (AP). Which activities are appropriate for the nurse to safely delegate? Select all that apply. A. Performing patient assessments B. Making patient beds C. Giving patients bed baths D. Administering oral medications E. Ambulating patients F. Assisting patients with meals

b, c, e, f. AP assist the RN to provide care as delegated by and under their supervision. Typical tasks delegated include actions for stable patients (e.g., vital signs hygiene, bed-making, ambulating patients, and helping to feed patients). Performing the initial patient assessment and administering medications are the responsibility of the RN.

A nurse working in an ambulatory care center provides care for patients experiencing varying levels of health and illness. The nurse bases care on which concepts of health and illness? Select all that apply. A. Health and illness are the same for all people. B. Health and illness are individually defined by each person. C. People with acute illnesses may consider themselves healthy. D. People with chronic illnesses have poor health beliefs. E. Health is more than the absence of illness. F. Illness is the response of a person to a disease.

b, c, e, f. Each person defines health and illness individually, based on a number of factors. Health is more than just the absence of illness; it is an active process in which a person moves toward their maximum potential. An illness is the response of the person to a disease

During morning huddle, a nurse manager and some nurses are identifying patients on the unit who are at risk for hospital-acquired infections (HAIs). Which patients will the nurses identify? Select all that apply. A. Smoker, two packs of cigarettes daily B. White blood cell count of 2,000/mm3 C. Indwelling urinary catheter in place D. Vegetarian and slightly underweight E. Central venous catheter present F. Postoperative colostomy

b, c, e, f. Leukopenia (low white blood cell count), indwelling urinary catheters, central venous catheters, and surgeries in which the wound is classified as dirty have been implicated in most HAIs. Cigarette smoking and a vegetarian diet have not been implicated as risk factors for HAIs.

An ambulatory care nurse serving a large, culturally diverse population is planning a free blood pressure screening clinic. Based on the nurse's understanding of racial differences in health and illness, which groups will the nurse target for screening? Select all that apply. A. Native American people B. African American people C. Alaska Native people D. Asian people E. White people F. Hispanic people

b, c, e. African American people, Asian people, and White people are more prone to developing hypertension. Alaska Native individuals and Native American individuals are prone to heart disease, diabetes, cirrhosis, and fetal alcohol syndrome.

A nurse uses critical-thinking skills to develop the care plan for an older adult with dementia awaiting placement in a long-term care facility. Which statements describe characteristics of the critical thinking used by nurses engaged in clinical reasoning? Select all that apply. A. Functions independently of nursing standards, ethics, and state practice acts B. Based on the principles of the nursing process, problem solving, and the scientific method C. Driven by patient, family, and community needs as well as nurses' needs to give competent, efficient care D. Avoids designs to compensate for problems created by human nature, such as medication errors E. Constantly reevaluating, self-correcting, and striving for improvement F. Focuses on the big picture rather than identifying the key problems, issues, and risks involved with patient care

b, c, e. Critical thinking applied to clinical reasoning and clinical judgment is guided by standards, policies and procedures, and ethics. When applying principles of nursing process, problem solving, and the scientific method, clinical reasoning identifies the key problems, issues, and risks. This is driven by patient, family, and community needs as well as nurses' needs to give competent, efficient care. It also calls for strategies that make the most of human potential and compensate for problems created by human nature. It is constantly reevaluating, self-correcting, and striving to improve the quality and safety of health care systems

A nurse answers a call light and finds the patient on the floor. After the health care provider examines the patient and finds no injury, the nurse returns the patient to bed and fills out an incident report. What statements are true about incident reports? Select all that apply. A. They can be used as disciplinary action against staff members. B. They can be used as a means of identifying risks. C. They can be used for quality control. D. They must be completed by the facility manager. E. They make facts available in litigation cases. F. They should be documented in the patient record.

b, c, e. Incident reports are used for quality improvement and should not be used for disciplinary action against staff members. They are a means of identifying risks and are filled out by the nurse responsible for the injured party. An incident report makes facts available in case litigation occurs; in some states, incident reports may be used in court as evidence. A health care provider completes the incident form with documentation of the medical examination of the patient, employee, or visitor with an actual or potential injury. Documentation in the patient record should not include the fact that an incident report was filed.

A nurse is assisting a postoperative patient with conditioning exercises to prepare for ambulation. Which instructions from the nurse are appropriate for this patient? Select all that apply. A. Do full-body pushups in bed six to eight times daily. B. Breathe in and out smoothly during quadricep-setting exercises. C. Place the bed in the lowest position or use a footstool for dangling. D. Dangle on the side of the bed for 30 to 60 minutes. E. Allow the nurse to bathe you completely to prevent fatigue. F. Perform quadriceps two to three times per hour, four to six times a day.

b, c, f. Breathing in and out smoothly during quadricep-setting exercises maximizes lung inflation. The patient should perform quadricep-setting exercises two to three times per hour, four to six times a day, or as ordered. The patient should never hold their breath during exercise drills because this places a strain on the heart. Pushups are usually done three or four times a day and involve only the upper body. Dangling for a few minutes is done to adjust to the upright position; dangling for 30 to 60 minutes is impractical for the nurse to supervise and may prove unsafe. The nurse should place the bed in the lowest position or use a footstool for dangling. The nurse should also encourage the patient to be as independent as possible to prepare for return to normal ambulation and ADLs.

A nursing student is writing a paper about nurses performing health promotion activities at the tertiary level. Which nursing actions will the student include? Select all that apply. A. Running an immunization clinic in a lower-income area of a city B. Teaching a patient with an amputation how to care for the residual limb C. Providing range-of-motion exercises for a patient who is paralyzed D. Teaching parents of toddlers how to childproof their homes E. Providing screening for scoliosis for school students F. Teaching new parents how to choose and use an infant car seat

b, c. Tertiary health promotion and disease prevention begins after an illness is diagnosed and treated to reduce disability and to help rehabilitate patients to a maximum level of functioning. Tertiary interventions include providing ROM exercises and patient teaching for residual limb care. Providing immunizations and teaching parents how to childproof their homes and use an appropriate car seat are primary health promotion activities. Providing screenings is a secondary health promotion activity.

A patient with an advanced directive and do-not-resuscitate order is sobbing and reporting severe pain. The nurse contacts the provider, who refuses to increase the medication dose due to the patient's hypotension. What actions would the nurse take next? Select all that apply. A. Lodge a complaint with the state board of nursing B. Consult with the ethics committee C. Contact a different health care provider D. Speak with the nurse manager E. Request a palliative care consultation

b, d, e. Ethical distress results from knowing the right thing to do but finding it almost impossible to execute due to institutional or other constraints (in this case, the nurse fears inability to collaborate with the provider). The nurse uses available resources such as seeking guidance from the nurse manager, attempting to secure a palliative care consult, and consulting the ethics committee. The state board of nursing does not regulate medical practice. The nurse does not circumvent the patient's health care provider.

A nurse on a mother-baby unit engages in informal planning while providing ongoing nursing care. What actions are included in this type of planning? Select all that apply. A. Sitting down with a patient and prioritizing existing diagnoses B. Assessing a woman for postpartum depression during patient education C. Planning interventions for a patient with a risk for bleeding D. Taking time to speak with a new mother who just received bad news E. Reassessing a patient who reports their pain medication is not working F. Coordinating home care for a patient being discharged later today

b, d, e. Informal planning is a link between identifying a patient's strength or problem and providing an appropriate nursing response, often while rearranging priorities. Examples of this include the nurse integrating assessment for postpartum depression during patient care, providing a therapeutic presence for a patient who received bad news, or reassessing a patient for pain during rounding. Formal planning involves prioritizing diagnoses, formally planning interventions, and coordinating the home care of a patient being discharged.

A nurse is planning teaching strategies in the affective domain of learning for patients with alcohol use disorders. Which teaching-learning activities will the nurse use? Select all that apply. A. Preparing a lecture on the harmful long-term effects of alcohol on the body B. Asking the patient to discuss reasons people with alcohol use disorders drink and exploring other methods of coping with problems C. Requesting that patients perform a return demonstration for using relaxation exercises to relieve stress D. Helping patients to reaffirm their feelings of self-worth and relate this to their alcohol use disorder E. Using a pamphlet to discuss the tenets of the Alcoholics Anonymous program with patients F. Reinforcing the mental benefits of gaining self-control over a substance use disorder

b, d, f. Affective learning includes changes in attitudes, values, and feelings (e.g., the patient expresses renewed self-confidence to be able to give up drinking). Cognitive learning involves the storing and recalling of new knowledge in the brain, such as the learning that occurs during a lecture or by using a pamphlet for teaching. Learning a physical skill involving the integration of mental and muscular activity is called psychomotor learning, which may involve a return demonstration of a skill.

A registered nurse is formulating nursing diagnoses for a patient with multiple fractures. Which actions does the nurse take during this step of the nursing process? Select all that apply. A. Conducting a nursing interview to collect patient data B. Analyzing data collected in the nursing assessment C. Developing a care plan for the patient D. Pointing out the patient's strengths E. Assessing the patient's mental status F. Identifying community resources to help the family cope

b, d, f. Diagnosing includes identifying actual or potential health problems for individuals, groups, or communities; identifying factors that contribute to or cause health problems (etiologies); and identifying resources or strengths the individual, group, or community can draw on to prevent or resolve problems. The nurse assesses and collects patient data in the assessment step and develops the care plan during the planning phase of the nursing process.

A nurse in a long-term care facility observes the AP providing foot care for patients. Which actions by the AP require the nurse to intervene? Select all that apply. A. Bathing the feet thoroughly in a mild soap and tepid water solution B. Soaking the resident's feet in warm water and bath oil C. Drying the feet and area between the toes thoroughly D. Applying an alcohol rub for odor and dryness to the feet E. Applying an antifungal foot powder F. Cutting the toenails at the lateral corners when trimming the nail

b, d, f. The nurse corrects the AP for soaking the feet or using alcohol and reminds them to use moisturizer if the feet are dry. Digging into or cutting the toenails at the lateral corners when trimming the nails requires correction; toenails should be trimmed straight across. Guidelines for foot care include bathing the feet thoroughly in a mild soap and tepid water solution; drying feet thoroughly, including the area between the toes; and applying an antifungal foot powder when requested.

A nursing student is performing hand hygiene after providing care to a patient who is in isolation for C. diff related to antibiotic therapy. Which actions by the nursing student will the primary nurse need to correct? Select all that apply. A. Removing all jewelry including a platinum wedding band B. Decontaminating the hands with an alcohol-based hand sanitizer C. Using approximately 1 teaspoon of liquid soap D. Keeping hands higher than elbows when placing under the faucet E. Using friction motion when washing for at least 20 seconds F. Rinsing thoroughly with water flowing toward the fingertips

b, d. After caring for patients with C. diff infection, proper handwashing includes using soap and water, then rinsing thoroughly with water flowing toward fingertips. Proper hand hygiene permits a plain wedding band to be worn; other jewelry is removed. The nurse uses about 1 teaspoon (5 mL) of liquid soap, using friction motion for at least 20 seconds, washing to 1 inch above the wrists using friction.

A nurse is prioritizing nursing care for patients on a medical-surgical unit. Which nursing interventions address patients' physiologic needs? Select all that apply. A. Preventing falls during admission B. Administering oxygen to a patient with shortness of breath C. Providing a magazine for a patient without visitors D. Assisting a patient who had a stroke eat their dinner E. Facilitating a visit from the patient's significant other F. Referring a patient to a cancer support group

b, d. Physiologic needs—oxygen, water, food, elimination, temperature, sexuality, physical activity, and rest—must be met at least minimally to maintain life. Providing food and oxygen are examples of interventions to meet these needs. Preventing falls helps meet safety and security needs; providing art supplies may help meet self-actualization needs; facilitating visits from loved ones helps meet self-esteem needs; and referring a patient to a support group helps meet love and belonging needs.

A nurse who is working in a hospital setting uses value clarification to help patients understand the values that motivate patient behavior. What patient actions help the nurse determine if they demonstrate "prizing" during this process? Select all that apply. A. They stop smoking after a diagnosis of lung cancer. B. They show off a new outfit that after losing 20 lb. C. They choose to work fewer hours following a myocardial infarction. D. They adopt a low-cholesterol diet. E. They join a gym and schedule classes throughout the year. F. They proudly display a certificate for completing a marathon.

b, f. Prizing something you value involves pride, happiness, and public affirmation, such as losing weight or running a marathon. When choosing, you choose freely from alternatives after careful consideration of the consequences of each alternative, such as quitting smoking and working fewer hours. Finally, the person who values something acts on the value by combining choice and behavior with consistency and regularity, such as joining a gym for the year and following a low-cholesterol diet faithfully.

A nurse develops a contractual agreement with a morbidly obese patient to achieve optimal weight goals. Which statement best describes the nature of this agreement? A. "This agreement forms a legal bond between the two of us to achieve your weight goals." B. "This agreement will motivate the two of us to do what is necessary to meet your weight goals." C. "This agreement will help us determine what learning outcomes are necessary to achieve your weight goals." D. "This agreement will limit the scope of the teaching session and make stated weight goals more attainable."

b. A contractual agreement is a pact two people make, setting out mutually agreed-on goals. Contracts are usually informal and not legally binding. When teaching a patient, such an agreement can help motivate both the patient and the teacher to do what is necessary to meet the patient's learning outcomes. The agreement notes the responsibilities of both the teacher and the learner, emphasizing the importance of the mutual commitment.

A nurse is updating the plan of care with nurse-initiated interventions. Which intervention is appropriate to include? A. Administering acetaminophen for a headache B. Offering emotional support to a patient C. Consulting with a physical therapist D. Attending a team meeting for care planning

b. A nurse-initiated intervention is related to the nursing diagnosis and projected outcome. It is an autonomous action based on scientific rationale. The physician or health care provider uses a physician-initiated interventions or order in response to the medical diagnosis: nurses execute these interventions safely and effectively. Collaborative interventions are initiated by other providers including pharmacists, respiratory therapists, or physician assistants.

After an initial skin assessment, the nurse documents the presence pressure area that is reddened and has a 1-cm blister. How will the nurse document the wound stage? A. Stage 1 dark maroon wound, skin intact B. Stage 2 with 1-cm blister noted C. Stage 3 wound base with red granulation tissue D. Stage 4 blanchable reddened area, 2 cm

b. A stage 2 pressure injury involves partial-thickness loss of dermis and presents as a shallow open ulcer with a red-pink wound bed, without slough. It may also present as an intact or open/ruptured serum-filled blister. Dark maroon or purple wounds with intact skin represent deep tissue injury. Red granulation tissue is present in stage 3 or 4 pressure injuries that are healing. A blanchable, red area is a stage 1 pressure injury.

A nurse on a medical-surgical unit is caring for a group of patients. For which patient will the nurse perform a focused assessment? A. Newly admitted B. Recent application of a wrist cast C. Signs of acute respiratory distress D. Post-abdominal surgery without complications

b. After application of a cast, the nurse performs a focused neurovascular assessment, to assess circulation, sensation, and motor ability. A newly admitted patient requires a comprehensive assessment. The nurse performs an emergency assessment on a patient who presents with signs of acute respiratory difficulty. A postoperative patient without complications will receive ongoing assessments at regular intervals to evaluate the effectiveness of care and to assess for new problems.

When developing the admission care plan for a patient with multiple sclerosis and quadriplegia, the nurse formulates the patient problem: Impaired Tissue Integrity: Impaired Skin Integrity. What action will the nurse take next? A. Elevate the patient's heels off the bed using a pillow B. Develop a goal that the patient will consume protein at each meal C. Delegate assessment of the skin on the patient's back to the AP D. Teach the patient to turn themselves in bed every hour

b. After the health problem is developed, the nurse begins the planning phase of the nursing process, which includes goal development. Elevating the heels and teaching the patient to turn are interventions used during the implementation phase. Delegating an assessment to the AP is an incorrect activity; assessment falls within the role and scope of practice of the professional nurse. In addition, the nurse must perform the skin assessment to develop the problem and plan care.

A nurse assisting a patient with a bed bath observes the older adult has dry skin, which the patient states is "itchy." Which intervention is appropriate? A. Bathe the patient more frequently. B. Use an emollient on the dry skin. C. Explain that this is expected as people age. D. Limit the patient's fluid intake.

b. An emollient soothes dry skin, whereas frequent bathing increases dryness. Telling the patient this is normal with aging and does not help resolve the issue. Limiting fluid intake can promote dehydration and exacerbate dry skin.

Nursing students have been assigned to discuss a section of a research article examining 24-hour visitation for patients in the intensive care unit (ICU). What topic will the student assigned to the applicability section plan to discuss? A. Description of the tool that was used to gather the data B. How the results could be used in the student's practice C. Statistical methods used to determine the results D. Whether informed consent was obtained from the participants

b. Applicability refers to how study results could be used in practice. Method includes the design and data analysis. Informed consent affirms the patient's right to agree to participate in a study without coercion, to refuse to participate without jeopardizing their care, the right to confidentiality, the right to be protected from harm, and the ability to withdraw from the study at any time.

The nurse reports to their manager that informed consent was not obtained from a patient for whom HIV testing was already performed. The nurse suggests which intentional tort may have been committed? A. Assault B. Battery C. Invasion of privacy D. False imprisonment

b. Assault is a threat or an attempt to make bodily contact with another person without that person's consent. Battery is an assault that is carried out. Every person is granted freedom from bodily contact by another person unless consent is granted. The Fourth Amendment gives citizens the right of privacy and the right to be left alone; a nurse who disregards these rights is guilty of invasion of privacy. Unjustified retention or prevention of the movement of another person without proper consent can constitute false imprisonment.

To plan the day, a nurse is prioritizing patient diagnoses according to Maslow's hierarchy of human needs. What patient problem will the nurse address first? A. Altered body image perception B. Impaired gas exchange C. Grief D. Situational low self-esteem

b. Because basic needs must be met before a person can focus on higher ones, Maslow's hierarchy of needs sets the priorities as: (1) physiologic needs, (2) safety needs, (3) love and belonging needs, (4) self-esteem needs, and (5) self-actualization needs. Answer (b) is an example of a physiologic need, (a and d) are examples of a self-esteem need, and (c) is an example of a love and belonging need.

A nurse enters the patient's room to perform pin-site care for a patient wearing a halo vest to stabilize the cervical spine. What action will the nurse take first? A. Administer pain medication B. Reassess the patient C. Prepare the equipment D. Explain the procedure to the patient

b. Before implementing any nursing action, the nurse returns to the first step of the nursing process, reassessing whether the action is still needed. Then the nurse may collect the equipment, explain the procedure, and, if necessary, administer pain medications.

A nurse provides care for postoperative patients using meticulous hand hygiene and aseptic technique. Which of Maslow's basic human needs is the nurse addressing? A. Physiologic B. Safety and security C. Self-esteem D. Love and belonging

b. By using meticulous hand hygiene and aseptic technique, nurses prevent infection, which falls under safety. An example of a physiologic need is clearing a patient's airway. Self-esteem needs may be met by allowing an older adult to talk about a past career. An example of helping meet a love and belonging need is contacting a hospitalized patient's family to arrange a visit.

A nurse is developing a clinical outcome for a patient who is an avid runner and is recovering from a stroke resulting in right-sided paresis. Which clinical outcome is most appropriate to include in the care plan? A. After receiving 3 weeks of physical therapy, patient will demonstrate improved movement on the right side of her body. B. By 8/15/25, patient will be able to use right arm to dress, comb hair, and feed herself. C. Following physical therapy, patient will begin to gradually participate in walking/running events. D. By 8/15/25, patient will verbalize feeling sufficiently prepared to participate in running events.

b. Clinical outcomes describe the expected status of health issues at certain points in time, after treatment is complete. Functional outcomes (b) describe the person's ability to function in relation to the desired usual activities. Quality-of-life outcomes (c) focus on key factors that affect someone's ability to enjoy life and achieve personal goals. Affective outcomes (d) describe changes in patient values, beliefs, and attitudes.

A patient experiencing chest pain asks the nurse why a nitroglycerin tablet must be placed under their tongue instead of swallowed. Which answer by the nurse is appropriate? A. "We could put the tablet between the cheek and gum, instead." B. "The area is rich in superficial blood vessels, and helps with absorption." C. "Swallowing interferes with quick systemic effects." D. "This is an enteric-coated tablet, designed for absorption outside the stomach."

b. Drugs with extensive or variable first-pass effects, such as nitroglycerin, are not given orally because most of the drug would be destroyed by the liver, with little or no drug left to work in the body. Sublingual medications should not be swallowed, but rather held in place so that complete absorption can occur. Before administering a sublingual or buccal drug, offer the patient water (if permitted) or oral care (if NPO). Placing a tablet between the cheek and gum is consistent with the buccal route of administration, not sublingual. Enteric-coated tablets are designed to dissolve after passing through the stomach.

An outbreak of measles has occurred at the local elementary school. The parents of a child in the prodromal phase of the illness are told the child should stay home until well. What is important for the nurse to teach the parents about the prodromal phase? A. The organisms enter the body and multiply while the patient is asymptomatic. B. A person typically has vague, nonspecific symptoms and is highly contagious. C. The presence of infection-specific signs and symptoms develop, manifesting as local or systemic responses. D. The signs and symptoms of the illness disappear, and the person returns to their preillness state.

b. During the prodromal stage, the person has vague signs and symptoms, such as fatigue and a low-grade fever. There are no obvious symptoms of infection during the incubation period, and symptoms are more specific and apparent during the full stage of illness, disappearing in the convalescent period.

A nurse says to their nurse manager, "I need the day off, and you didn't give it to me!" The manager replies, "I wasn't aware you needed the day off, and it isn't possible since staffing is inadequate." How could the nurse best modify the communication for a more positive interaction? A. "I placed a request to have 8th of August off for a doctor's appointment, but I'm scheduled to work." B. "Could I make an appointment to discuss my schedule with you? I requested the 8th of August off for a doctor's appointment." C. "I will need to call in on the 8th of August because I have a doctor's appointment." D. "Since you didn't give me the 8th of August off, will I need to find someone to work for me?"

b. Effective communication involves sending clear, nonthreatening, and respectful information to the receiver. The nurse identifies the subject of the meeting and determines a mutually agreed upon time.

A nurse is conducting qualitative research to study the culture of Native Alaskans and how their diet affects their overall state of health. Which method of research is the nurse using? A. Historical B. Ethnography C. Grounded theory D. Phenomenology

b. Ethnographic research was developed by the discipline of anthropology and is used to examine issues of culture of interest to nurses. Historical research examines events of the past to increase understanding of the nursing profession today. The basis of grounded theory methodology is the discovery of how people describe their own reality and how their beliefs are related to their actions in a social scene. The purpose of phenomenology (both a philosophy and a research method) is to describe experiences as they are lived by the subjects being studied.

A nurse in the emergency department is using the Glasgow coma scale to assess a patient who was struck in the head and upper body with a baseball bat. Based on the information in the neurologic assessment, what numerical value will the nurse assign? (EHR) Neurologic assessment: The patient opens his eyes to painful stimulus, does not speak, and withdraws to painful stimulus. Unable to assess orientation to person, time, and place. A. 3 B. 7 C. 11 D. 15

b. Eye opening to painful stimulus = 2 points, no speech = 1 point; and withdrawal to painful stimulus = 4 points, for a total score of 7. A score of 8 or less is associated with coma.

A nurse is performing an initial comprehensive assessment of a patient admitted to a long-term care facility. The nurse begins the assessment by asking the patient, "How would you describe your health status and well-being?" and, "What do you do to keep yourself healthy?" These questions reflect what model for organizing data? A. Maslow's hierarchy of needs B. Gordon's functional health patterns C. Human response patterns D. Body system model

b. Gordon's functional health patterns begin with the patient's perception of health and well-being and progress to data about nutritional-metabolic patterns, elimination patterns, activity, sleep/rest, self-perception, role relationship, sexuality, coping, and values/beliefs. Maslow's model is based on the human needs hierarchy. Human responses include exchanging, communicating, relating, valuing, choosing, moving, perceiving, knowing, and feeling. The body system model is based on the functioning of the major body systems.

A clinic nurse works with a population experiencing health disparities. Which patient does the nurse identify as having the highest risk for postponing annual breast examinations and mammograms? A. Patient whose best friend had a benign breast lump removed B. Patient who lives in a low-income neighborhood C. Patient who has a family history of breast cancer D. Patient whose family encourages regular physical examinations

b. Healthy People 2020 defines health disparity as a "particular type of health difference that is closely linked with social, economic, and/or environmental disadvantage." Disparities are influenced by race and ethnicity, poverty, sex assigned at birth, age, mental health, educational level, disabilities, sexual orientation, health insurance, and access to health care.

A nursing student asks the primary nurse why an immobile patient developed two urinary tract infections (UTIs) in the 6 months. How does the nurse best explain this patient's risk for UTI? A. Improved renal blood supply to the kidneys B. Urinary stasis C. Decreased urinary calcium D. Acidic urine formation

b. In a nonerect patient, the kidneys and ureters are level, limiting or delaying urinary drainage from the kidney pelvis to ureter and bladder. The resulting urinary stasis favors the growth of bacteria that can promote urinary tract infections. Regular exercise, not immobility, improves blood flow to the kidneys. Immobility predisposes the patient to bone demineralization, resulting in increased urinary calcium levels and alkaline urine, contributing to renal calculi and urinary tract infection, respectively.

A nurse caring for families in a free health care clinic assesses for psychosocial risk factors for altered family health. Which example best describes one of these risk factors? A. The family does not have dental care insurance or resources to pay for it. B. Both parents work and leave a 12-year-old child to care for his younger brother. C. Both parents and their children are considerably overweight. D. The youngest member of the family has cerebral palsy and needs assistance from community services.

b. Inadequate childcare resources are a psychosocial risk factor. Not having access to dental care and obese family members are lifestyle risk factors. Having a family member with birth defects is a biologic factor.

The nursing philosophy in an acute care hospital includes a commitment to deliver thoughtful, person-centered care. Which description of the nursing process best supports this commitment? A. Systematic B. Interpersonal C. Dynamic D. Universally applicable in nursing situations

b. Interpersonal. All other options are characteristics of the nursing process but focus on the patient best illustrates the interpersonal dimension of the nursing process.

Nurses in a long-term care facility use Maslow's hierarchy of basic human needs to plan care for their patients. What is the expected outcome when using this hierarchy? A. Accurate nursing diagnoses B. Clear priorities of care C. Concerns communicated concisely D. Integration of science into nursing care

b. Maslow's hierarchy of basic human needs is useful for establishing priorities of care.

A home health nurse teaches a patient to a change the dressing for a chronic venous stasis ulcer using clean technique. Which principle of asepsis will the nurse consider when preparing the teaching plan? A. The nurse chooses clean or sterile technique based on personal preference. B. The use of clean technique is considered safe in the home setting. C. Surgical asepsis is the safest method to use in a home setting. D. The patient can use clean technique; their partner must wear sterile gloves.

b. Medical asepsis, or clean technique, involves procedures and practices that reduce the number and transfer of pathogens. This is usually recommended in the home setting, where the patient's environment is more controlled. Injections require surgical asepsis. The patient and partner share the same home; medical asepsis is appropriate.

A nurse on a medical-surgical unit is planning to administer an antibiotic to a patient with a kidney infection who is 10 weeks' pregnant. The drug reference states that the medication is teratogenic. Which action will the nurse take? A. Administer the medication, because the risk of illness is greater than the benefit of the medication. B. Hold the medication and collaborate with the health care provider to find an alternative. C. Ask the patient if they consent to receive the medication and document the response in the electronic health record. D. Collaborate with the pharmacist on dose reduction.

b. Medications that are identified as teratogenic have the potential to cause developmental defects in the embryo or fetus and are contraindicated. The nurse must hold the medication and notify the health care provider of the pregnant person's pregnancy status. The patient is not the best person to determine the impact of this medication on the fetus; the medication is a teratogen, even with dose reduction.

An RN in a long-term care facility supervises APs as they provide hygiene to older adults. What action by the AP will the nurse correct? A. When providing perineal care, washing the area from front to back B. Insisting the older adult must take a bath or shower each day C. Telling the patient to avoid soaking feet, helps the patient dry between the toes D. Covering areas not being bathed with a bath blanket

b. Older adults tend to develop dry skin; bathing frequency will change accordingly. Soaking adults' feet is not recommended. It is appropriate to keep a patient warm with bath blankets and provide perineal care washing from front to back.

A nursing student is performing a nursing history for the first time. The student asks the primary nurse how anyone learns all the questions needed to get complete baseline data. What would be the nurse's best reply? A. "There's a lot to learn at first, but once it becomes part of you, you just ask the same questions over and over in each situation until you can do it in your sleep!" B. "You make the basic questions a part of you and apply critical thinking to modify them, to help you plan quality care." C. "It is really hard to learn how to do this well, as each history is different. I often feel like I'm starting fresh with each new patient." D. "Don't worry about learning all of the questions to ask. Every facility has its own assessment form you must use."

b. Once a nurse learns what constitutes the minimum data set, it can be adapted to each patient situation. It is not true that each assessment is the same even when using the same minimum data set, nor is it true that each assessment is uniquely different. Nurses committed to thoughtful, person-centered practice individualize their questions to each patient and situation. When using a standard facility assessment tool the nurse must still use critical thinking to individualize questions or follow up on patient information.

As part of a hospital-wide quality-assurance program, an electronic medical record review for the last 6 months reveals a higher incidence of falls on a specific unit. The nurse authoring the study refers to the review as what type of evaluation? A. Quality by inspection B. Retrospective evaluation C. Concurrent study D. Quality by indicator

b. Quality by inspection focuses on finding deficient workers and removing them. Concurrent evaluation uses direct observation of nursing care, patient interviews, and chart review to determine whether the specified evaluative criteria are met. Retrospective evaluation may use post discharge questionnaires, patient interviews (by telephone or face to face), or chart review (nursing audit) to collect data. Quality as opportunity focuses on finding opportunities for improvement and fosters an environment thriving on teamwork, with people sharing the skills and lessons they have learned.

Nurses note that allowing patients to choose the time of their breakfast to improve patient satisfaction has resulted in medication delays for patients who have prescriptions for medications taken on an empty stomach. Which action will direct the nurses to the best outcome? A. Asking the pharmacy to dispense the medication at bedtime B. Suggesting a quality improvement project piloting a 6:00 AM administration C. Requesting that the health care provider prescribe the medication for midnight D. Telling the nurse manager that patients are getting their medications late

b. Quality improvement or continuous quality improvement involves systematic, continuous actions that lead to measurable improvement in health care services and the health status of targeted patient groups. Quality-assurance programs enable nursing to be accountable for the quality of nursing care. Making changes without gathering needed data may prove unsafe or a waste of time. Reporting to the nurse manager does not reflect professional commitment to improving processes.

A nurse notes a pressure wound base is red. Using the RYB system for documentation, what intervention is indicated? A. Irrigating the wound and applying an absorbent dressing B. Gently cleansing the wound and applying a moist dressing C. Discussing consultation for surgical debridement with the provider D. Performing frequent dressing changes to keep the wound and dressing dry

b. Red wounds are in the proliferative stage of healing and reflect the color of normal granulation tissue. Wounds in this stage need protection with nursing interventions that include gentle cleansing, use of moist dressings, and dressing changes only when necessary (or based on product manufacturer's recommendations). To cleanse yellow wounds, nursing interventions include the use of wound cleansers and irrigation. The eschar found in black wounds requires debridement (removal) before the wound can heal.

What is the correct procedure to carry out this prescription? MAR 7:30 AM: 40 units of NPH insulin and 10 units of regular insulin daily subcutaneously. A. Inject air into the regular insulin vial and withdraw 10 units; then, using the same syringe, inject air into the NPH vial and withdraw 40 units of NPH insulin. B. Inject air into the NPH insulin vial, being careful not to allow the solution to touch the needle; next, inject air into the regular insulin vial and withdraw 10 units; followed by withdrawal of 40 units of NPH insulin. C. Inject air into the regular insulin vial, being careful not to allow the solution to touch the needle; next, inject air into the NPH insulin vial and withdraw 40 units; then, withdraw 10 units of regular insulin. D.Inject air into the NPH insulin vial and withdraw 40 units; then, using the same syringe, inject air into the regular insulin vial and withdraw 10 units of regular insulin.

b. Regular or short-acting insulin (unmodified insulin) should never be contaminated with NPH or any insulin modified with added protein. Placing air in the NPH vial first without allowing the needle to contact the solution ensures that the regular insulin will not be contaminated.

A young Hispanic mother comes to the local clinic because her baby is sick. She speaks only Spanish, and the nurse speaks only English. Which action should the nurse take next? A. Use short words and speak loudly B. Obtain a medical interpreter C. Explain why care cannot be provided D. Provide instructions in writing

b. Requesting an interpreter reflects best practice. Qualified interpreters, available in many facilities or via video or phone, have knowledge of health care and can provide assistance. Using short words, talking loudly, and providing instructions in writing will not aid communication. Explaining why care cannot be provided will not meet the patient's health needs; the nurse is required to provide care. Since the patient doesn't speak English, this will not be understood.

A community health nurse cares for vulnerable populations. What problem will the nurse prioritize for a patient who is homeless? A. Love and belonging B. Safety C. Self-esteem D. Self-actualization

b. Safety is the most appropriate focus as adults and children experiencing homelessness do not have a regular and appropriate place to sleep; they may be sheltered or unsheltered. After meeting safety needs, the nurse can focus on love and belonging, self-esteem, or self-actualization

A nurse on a surgical unit has assessed and documented a patient's wound and drainage. Which statements most accurately describe the characteristic of the wound drainage? Graphic Record T 99.9 P100 RR 20 BP 138/88 Nursing note: Patient postoperative day 2. Dry sterile dressing changed on abdominal incision. Incision edges are well approximated with a slight ½-cm opening at inferior edge; incisional edges reddened. Hemovac draining sanguineous material, 60 mL for the shift. Patient reports moderate pain, relieved by oxycodone X1. A. Sanguineous drainage is composed of the clear portion of the blood and serous membranes. B. Sanguineous drainage is composed of a large number of red blood cells and looks like blood. C. Sanguineous drainage is composed of white blood cells, dead tissue, and bacteria. D. Sanguineous drainage is thin, cloudy, and watery and may have a musty or foul odor.

b. Sanguineous drainage consists of large numbers of red blood cells and looks like blood. Bright-red sanguineous drainage is indicative of fresh bleeding, whereas darker drainage indicates older bleeding. Serous drainage, generally watery, is composed primarily of the clear, serous portion of the blood and serous membranes. Purulent drainage is made up of white blood cells, liquefied dead tissue debris, and both dead and live bacteria. It is thick, often has a musty or foul odor, and varies in color (such as dark yellow or green), depending on the causative organism.

A nursing student questions the primary nurse about instilling air into a nasogastric tube to confirm placement, when they learned that x-ray validation of the tube's tip in the stomach reflects best practice. The student is validating safe nursing practice with which type of knowledge? A. Instinctive knowledge B. Scientific knowledge C. Authoritative knowledge D. Traditional knowledge

b. Scientific knowledge is obtained through the scientific method or research; this leads to evidence-based practice. Instinct, such as. "I feel this is correct," is not a source of knowledge. Traditional knowledge is the part of nursing practice passed down from generation to generation, often without research data to support it. Examples include daily bathing and changing bed linens each day. Authoritative knowledge comes from an expert and is accepted as truth based on the person's perceived expertise.

A nurse is caring for a patient who is admitted to the hospital with traumatic injuries sustained in a motor vehicle accident. While hospitalized, the patient's spouse tells the patient that their house flooded, damaging their belongings. When the nurse notes that the patient is visibly upset by this news, the nurse suggests which type of counseling? A. Long-term developmental B. Short-term situational C. Short-term motivational D. Long-term motivational

b. Short-term counseling might be used during a situational crisis, which occurs when a patient faces an event or situation that causes a disruption in life, such as a flood. Long-term counseling extends over a prolonged period; a patient experiencing a developmental crisis, for example, might need long-term counseling. Motivational interviewing is an evidence-based counseling approach that involves discussing feelings and incentives with the patient. A caring nurse can motivate patients to become interested in promoting their own health.

A nursing student is preparing to administer medications and asks the clinical instructor about legal liability in clinical practice. What is the most appropriate response? A. "Students are not responsible for their acts of negligence resulting in patient injury." B. "Student nurses are held to the same standard of care that would be used to evaluate the actions of a registered nurse." C. "Hospitals are exempt from liability for student negligence if the student nurse is properly supervised by an instructor." D. "Most nursing programs carry group professional liability making student personal professional liability insurance unnecessary."

b. Student nurses are held to the same standard of care that would be used to evaluate the actions of a registered nurse. Student nurses are responsible for their own acts of negligence if these result in patient injury. A hospital may also be held liable for the negligence of a student nurse enrolled in a hospital-controlled program because the student is considered an employee of the hospital. Nursing instructors may share responsibility for damages in the event of patient injury if an assignment called for clinical skills beyond a student's competency or the instructor failed to provide reasonable and prudent clinical supervision. Most nursing programs require students to carry personal professional liability insurance.

A nurse is teaching first aid to counselors of a summer camp for children with asthma. This is an example of what aim of health teaching? A. Promoting health B. Preventing illness C. Restoring health D. Facilitating coping

b. Teaching first aid is a function of the goal to prevent illness. Promoting health involves helping patients to value health and develop specific health practices that promote and maintain wellness. Restoring health occurs once a patient is ill, and teaching focuses on developing self-care practices that promote recovery. When facilitating coping, nurses help patients come to terms with whatever lifestyle modification is needed for their recovery or to enable them to cope with permanent health alterations.

A nurse caring for patients in the city-run health clinic expresses a commitment to social justice. Working toward which action best exemplifies this attribute? A. Providing honest information to patients and the public B. Promoting universal access to health care C. Planning care in partnership with patients D. Documenting care accurately and honestly

b. The American Association of Colleges of Nursing lists promoting universal access to health care as an example of social justice. Providing honest information and documenting care accurately and honestly are examples of integrity, and planning care in partnership with patients is an example of autonomy.

A nurse is using the Explanatory Model of Health and Illness (ESFT) model to assess how a patient from another culture views their diagnosis of chronic obstructive pulmonary disease (COPD). What interview question is most appropriate to assess the E aspect of this model? A. How do you get your medications? B. How does having COPD affect your lifestyle? C. Are you concerned about the side effects of your medications? D. Can you describe how you will take your medications?

b. The ESFT model, a cross-cultural communication tool, guides providers in understanding a patient's explanatory model (a patient's conception of their illness), social and environmental factors, and fears and concerns and also guides providers in contracting for therapeutic approaches. Asking the questions: "How does having COPD affect your lifestyle?" explores the explanatory model, "How do you get your medications?" refers to the social and environmental factor, "Are you concerned about the side effects of your medications?" addresses fears and concerns, and "Can you describe how you will take your medications?" involves therapeutic contracting.

At an interprofessional meeting, nurses discuss appropriate uses for an organizational ethics committee. Which referral do the nurses identify as appropriate? A. Giving input into policies affecting work life balance B. Providing interprofessional input on clinical care C. Determining if the patient's cultural beliefs are valid D. Advising the board of nursing on policy for licensure

b. The Joint Commission mandates that accredited facilities have a mechanism for addressing ethical problems. Health care institutions often have multidisciplinary ethics committees who provide case review and consultation and participate in education, policy making, quality, and (in some cases) research. Nurses contribute unique knowledge about the patient and family, interpret technical facts, identify appropriate decision makers, represent the patient's best interests, and help ensure the course of action is justified by sound ethical principles. Nurses play an important role in policy making, identifying needed policies to address ethical concerns or suggest needed modifications of existing policies. The human resource department focuses on employee policy; the health care team cannot judge or force a patient to reevaluate their cultural beliefs and observances.

Nurses of a statewide nursing organization come together to plan for the challenges to health care for the 21st century. What current trend in health care should be a focus of the plan? A. Decreasing numbers of hospitalized patients B. Increasing numbers of older and more acutely ill patients C. Decreasing health care costs owing to managed care D. Decreasing advances in medical knowledge and technology

b. The National Advisory Council on Nurse Education and Practice identifies the following critical challenges to nursing practice in the 21st century: A growing population of hospitalized patients who are older and more acutely ill, increasing health care costs, and the need to stay current with rapid advances in medical knowledge and technology.

Nurses use the Nursing Interventions Classification (NIC) Taxonomy structure as a resource to plan nursing care for patients. What information is found in this structure? A. Case studies illustrating a complete set of activities that a nurse performs to carry out nursing interventions B. Nursing interventions, each with a label, a definition, and a set of activities that a nurse performs to carry it out, with a short list of background readings C. Complete list of nursing diagnoses, outcomes, and related nursing activities for each nursing intervention D. Complete list of reimbursable charges for each nursing intervention

b. The Nursing Interventions Classification (NIC) Taxonomy lists nursing interventions, each with a label, a definition, a set of activities that a nurse performs to carry it out, and a short list of background readings. It does not contain case studies, diagnoses, or charges.

When caring for a patient who sustained a spinal cord injury, the nurse formulates the health problem: Impaired Tissue Integrity Etiology: sensory and motor deficit Signs and symptoms: difficulty turning, reddened areas on heels and sacrum Which phrase gives direction to the underlying cause of the problem? A. Impaired Tissue Integrity B. Sensory and motor deficit C. Signs and symptoms D. Reddened areas of skin on the heels and back

b. The etiology, sensory and motor deficits, identifies the contributing or causative factors of the problem. The problem, "Impaired Tissue Integrity: Impaired Skin Integrity," states the undesirable health condition, life processes, or human response. The phrase, "Signs and symptoms: non-blanchable reddened areas on heels and back," contains the defining characteristics of the problem.

A nurse and AP are caring for a patient who just returned from the operating room after a femoral-popliteal arterial bypass graft. The nurse is getting another admission. What activity can the nurse safely delegate to the AP? A. Determining if pedal pulses are present B. Evaluating the patient's pain C. Reinforcing the sterile dressing D. Ordering dressing supplies

b. The nurse can delegate noncomplex activities to the AP such as obtaining (dressing) supplies, bedmaking, bathing, I & O, toileting, and ambulation. The nurse must perform assessments, provide teaching, perform sterile procedures, and develop the care plan.

The nurse has taught a patient with diabetes how to administer subcutaneous insulin injections. Which is the best strategy to evaluate if the teaching goal has been met? A. Ask the patient the insulin dose and times of day they will administer insulin. B. Observe the patient's technique in drawing up and administering insulin. C. Have the patient explain the skill they have just learned. D. Document the teaching session in the patient's electronic health record.

b. The nurse cannot assume that the patient has actually learned the content unless there is some type of proof of learning. The key to evaluation is ensuring the learner meets the outcomes stated in the teaching plan, in this case, by demonstrating the psychomotor skill.

A nurse notices a patient crying after meeting with the health care provider. Prior to formulating a health problem of difficulty coping, the nurse seeks to further support the problem by gathering which data? A. Abnormal vital signs B. Underlying cause of the tears C. Admitting diagnosis D. Patient's support system

b. The nurse continues gathering data, determining the presence of a problem of grief, impaired coping, etc., by determining the underlying cause of the tears. If the patient received news that a biopsy was free from cancer, perhaps no problem exists. If the patient was told they have a terminal illness, the nurse can continue to gather data and plan to support the patient's physical and emotional needs.

A visiting nurse is following up with a patient who was given a prescription for a diuretic and told to chart her weight daily. The patient's weight has increased 5 lb since the nurse's last visit. What actions will the nurse take first? A. Explain to the patient that it is clear she is not adhering to her prescription and the health care provider will be notified B. Document the 5-lb weight gain and ask the patient about sodium intake and medication side effects C. Terminate the plan of care while determining the cause for the weight gain D. Encourage the patient to continue the prescription and return in 1 week

b. The nurse documents the goal has not yet been achieved and also suspects the patient has not adhered to the prescription, perhaps due to frequent urination or other side effects. The nurse further assesses the patient's understanding of the medication's purpose and effects, understanding of the disease process and complications.

A nurse is planning to administer digoxin to a patient. After reviewing the medical record, what action will the nurse take? Electronic Health Record Prescriptions: -11/22/2025 digoxin loading dose 0.25 mg IV twice today only -11/23/2025 begin digoxin 0.125 mg orally daily Laboratory Studies: -Digoxin level: 2.7 ng/mL (reference range 0.5-2 ng/mL) A. Administer the medication, recording the level in the MAR. B. Hold the medication and confer with the prescriber. C. Give the patient one half the dose. D. Evaluate the patient's kidney function studies.

b. The nurse is responsible for safe medication administration, including interpreting therapeutic serum drug levels. The nurse withholds the medication based on the toxic (high) drug level. Next, the nurse collaborates with the health care provider to determine if a dose adjustment is indicated. Prescribing is outside the nurse's scope of practice; determining a half dose is needed is a prescription. The patient's renal function will be evaluated periodically, but it is most important not to give a medication when the blood level demonstrates toxicity.

A nursing student is preparing to administer morning care to a patient. What question by the student is most important to ask? A. "Would you prefer a bath or a shower?" B. "May I help you with a bed bath now or later this morning?" C. "I will be giving you your bath. Do you use soap or shower gel?" D. "I prefer a shower in the evening. When would you like your bath?"

b. The nurse should ask permission to assist the patient with a bath. This allows for patient preferences and consent for care that involves entering the patient's personal space.

A nurse receives a call from a friend requesting information on her mother-in-law who was just admitted to the hospital. How does the nurse best respond? A. "You shouldn't be asking me to do this. I could be fined or lose my job for disclosing this information." B. "I'm sorry; per privacy laws, I can't give out patient information—even to my best friend or a family member." C. "Because of HIPAA, you could get in trouble for asking for this information unless you are authorized by the patient to receive it." D. "Why are you asking? Are you extremely worried?"

b. The nurse should immediately clarify they must adhere to HIPAA laws to protect patient privacy and confidentiality. Mentioning penalties for breaches of privacy sidesteps the need to clearly introduce or reinforce the policy. It may be appropriate to ask the friend about her concerns, only after clarifying privacy laws.

When documenting a dressing change to a residual right limb, the nurse erroneously documents that the dressing change was performed on the left leg. How will the nurse most appropriately correct the documentation? A. Use white correction fluid to cover the error and neatly write over the correction B. Draw a single line through the error, write "mistaken entry," add correct information, and date and initial C. Blacken out the error with permanent marker and rewrite the note in the next available space D. Leave the entry in place, create a correctly written entry below, and cite the charting error above

b. The nurse should not black out, use erasers or correcting fluids to correct documentation, or remove a page with an error and rewrite the data on a new page. To correct an error after it has been entered, the nurse should mark the entry "mistaken entry," add the correct information, and date and initial the entry. If the nurse records information in the wrong chart, the nurse should write "mistaken entry—wrong chart" and sign off. The nurse should follow similar guidelines in electronic records.

A nursing student on the surgical unit is assigned to perform a review of systems using the head-to-toe format on a patient admitted for a fractured femur. Using this format, what system will the student assess first? A. Genitourinary B. Neurologic C. Respiratory D. Musculoskeletal

b. The nursing physical assessment involves the examination of all body systems in a systematic manner, commonly using a head-to-toe format called the review of systems (ROS). This assessment begins at the top of the body with the neurologic system and moves downward.

A nurse is caring for a patient with dehydration who has a prescription to encourage oral fluids. Which outcome statement will best direct nursing interventions? A. Offer patient 60 mL of fluid every 2 hours while awake. B. During the next 24-hour period, patient's fluid intake will total at least 2,000 mL. C. Teach the patient the importance of drinking enough fluids to prevent dehydration by 1/15/25. D. At the next visit on 12/23/24, patient will know to drink at least 3 L of water per day.

b. The outcomes in (a) and (c) make the error of expressing the patient goal as a nursing intervention. Incorrect: "Offer the patient 60 mL fluid every 2 hours while awake." Correct: "The patient will drink 60 mL fluid every 2 hours while awake, beginning 1/3/25." The outcome in (d) makes the error of using verbs that are not observable and measurable. Verbs to be avoided when writing outcomes include "know," "understand," "learn," and "become aware."

A patient has a fractured left leg, which has been casted. Following teaching from the physical therapist for using crutches, the nurse reinforces which teaching point with the patient? A. Lean on the crutches using the axillae to bear body weight. B. Keep elbows close to the sides of the body. C. When rising, extend the uninjured leg to prevent weight bearing. D. To climb stairs, place weight on affected leg first.

b. The patient should keep the elbows at the sides, prevent pressure on the axillae to avoid damage to nerves and circulation, extend the injured leg when rising to prevent weight bearing, and advance the unaffected leg first when climbing stairs.

A nurse is scheduling hygiene for patients on the unit. What is the priority the nurse uses to guide planning for patient's personal hygiene? A. When the patient had their most recent bath B. The patient's usual hygiene practices and preferences C. Where the bathing fits in the nurse's schedule D. The time that is convenient for the AP

b. The patient's preferences, practices, and rituals should always be taken into consideration, unless there is a clear threat to health. The patient and nurse should work together to come to a mutually agreeable time and method to accomplish the patient's personal hygiene. The availability of staff to assist may be important, but the patient's preferences are a higher priority.

A nurse is planning education for a patient who is scheduled for a diagnostic procedure to rule out a cerebral aneurysm. Which aspect of nursing does the nurse use as the basis for education? A. Art of nursing B. Science of nursing C. Caring aspect of nursing D. Holistic approach to nursing

b. The science of nursing is the knowledge base for care that is provided. In contrast, the skilled application of that knowledge is the art of nursing. Providing holistic care to patients based on the science of nursing is considered the art of nursing.

A nurse in the neurology clinic is assessing a patient's eyes for extraocular movements. Which correctly describes the procedure for this test? A. Ask the patient to sit about 3 ft away, facing the nurse. B. Have the patient follow a penlight held 1 ft from their face slowly through the cardinal positions. C. Move a penlight in concentric circular motion in front of the patient's eyes. D. Ask the patient to cover one eye with a hand or index card.

b. The steps in testing for extraocular movement are: (1) Ask the patient to sit or stand about 2 ft away, facing the nurse, who is sitting or standing at eye level with the patient; (2) ask the patient to hold the head still and follow the movement of a forefinger or a penlight with the eyes; (3) keeping the finger or light about 1 foot from the patient's face, move it slowly through the cardinal positions of gaze—up and down, left and right, diagonally up and down to the left, diagonally up and down to the right.

A nurse is writing outcomes for a patient admitted with a cardiac condition causing fluid overload and edema. Which reflects an appropriately worded outcome? A. Offer to elevate the patient's legs on a stool while out of bed B. Patient will restrict fluids to 1,500 mL per 24-hour period C. Monitor the patient's intake and output D. Weigh the patient each morning prior to breakfast

b. The terms goal, objective, and outcome are often used interchangeably to refer to the expected conclusion to the patient's health problem or expectation. Nurses use the phrase expected outcomes to refer to the more specific, observable, and measurable changes. Options a, c, and d are stated as interventions, rather than outcomes.

A nurse is using the Katz Index of Independence in Activities of Daily Living (ADLs) to assess the mobility of a hospitalized patient. During the patient interview, the nurse documents the following patient data: "Patient bathes self completely but needs help with dressing. Patient toilets independently and is continent. Patient needs help moving from bed to chair. Patient follows directions and can feed self." Based on this data, which score would the patient receive on the Katz index? A. 2 B. 4 C. 5 D. 6

b. The total score for this patient is 4. On the Katz Index of Independence in ADLs, one point is awarded for independence in each of the following activities: bathing, dressing, toileting, transferring, continence, and feeding.

A nurse is caring for a patient who is on bedrest following a spinal injury. Which action is appropriate to prevent foot drop? A. Maintain the supine position with supination on the feet. B. Ask the family to bring in high-top sneakers to maintain foot dorsiflexion. C. Encourage hyperextension of the feet with adaptive devices or splints. D. Use pillows to keep the feet in the abducted position.

b. To prevent foot drop, the nurse should support the feet in dorsiflexion using a footboard and/or high-top sneakers for further support. Supination involves lying patients on their back or facing a body part upward, and hyperextension is a state of exaggerated extension. Abduction involves lateral movement of a body part away from the midline of the body. These positions do not prevent foot drop.

A new home health nurse and preceptor are reviewing charting for a patient with advanced lung cancer who receives Medicare benefits. When reviewing a draft of the new nurse's documentation, which statement will the preceptor correct? A. Explained to family that irregular respirations or agitation may occur when the patient is actively dying B. Patient seemed in better spirits and reported going out for ice cream with his family yesterday C. Stage 3 pressure ulcer dressing on sacral area is dry and intact; due to be changed tomorrow D. Performed medication reconciliation with focus on pain management and anticoagulation

b. To receive Medicare services, the patient must be homebound, still needs skilled nursing care, or that the patient is dying, among others. Leaving the home for ice cream may interfere with home care benefits.

The nurse manager of a unit with an excellent safety record meets with staff to present the findings of a recent audit. The manager states, "We're doing well, but I believe we can do better. Who's got an idea to foster increased patient well-being and satisfaction?" This leader has demonstrated they value which process? A. Quality assurance B. Quality improvement C. Process evaluation D. Outcome evaluation

b. Unlike quality assurance, quality improvement (QI) is internally driven. QI focuses on patient care rather than organizational structure and processes rather than people, and has no end points. Its goal is improving quality rather than assuring quality. Process evaluation and outcome evaluation are types of quality-assurance programs.

A nurse is preparing to admit a patient with urinary sepsis related to vancomycin-resistant enterococci (VRE). While awaiting the patient's arrival, which of these actions will the nurse take? A. Prepare a negative-pressure room B. Ask the AP to get a supply of protective gowns C. Post a sign that visitors must wear a mask D. Obtain sterile gloves for personal care

b. VRE is spread via contact with the feces, urine, or blood of an infected or colonized person. Contact precautions, meticulous hand hygiene, reducing the use of invasive devices, environmental cleaning, and decolonizing high-risk patients are indicated. Some institutions have discontinued using contact precautions and use standard precautions, with no significant change in the incidence of MRSA or VRE.

The nurse is caring for a patient experiencing bronchospasm due to an exacerbation of asthma. During auscultation, the nurse anticipates the presence of which breath sound? A. Sibilant B. Wheezes C. Rhonchi D. Crackles

b. Wheezes are musical or squeaking high-pitched, continuous sounds heard as air passes through narrowed airways, such as with bronchospasm found in asthma or COPD. Rhonchi are low-pitched, continuous sounds with a snoring quality, which may clear with coughing; they occur when air passes through secretions. Crackles are discontinuous bubbling, cracking, or popping, low- to high-pitched sounds, that occur when air passes through fluid in the airways.

A nurse administering an injection to a patient who tested positive for HIV sustains a needlestick. What action should the nurse take first? A. Report the incident to the nurse manager and file an injury report B. Wash the exposed area with warm water and soap C. Consent to postexposure prophylaxis (PEP) at the appropriate time D. Set up counseling sessions regarding safe practice to protect self

b. When a needlestick injury occurs, the nurse should wash the affected area immediately with warm water and soap, report the incident to the nurse manager or appropriate person and complete an injury report, consent to and await the results of blood tests, consent to PEP, and attend counseling sessions regarding safe practice to protect self and others.

After assessing a patient recovering from a stroke in a rehabilitation facility, the nurse's initial analysis suggests a potential health problem of situational low self-esteem. How will the nurse record the problem when they believe more data are needed? A. No problem B. Possible problem C. Actual nursing diagnosis D. Clinical problem other than nursing

b. When a possible problem exists, the nurse must collect more data to confirm or disprove the suspected problem. The conclusion, "no problem," indicates no nursing response is required. When an actual problem is identified, the nurse continues using the steps of the nursing process by planning, goal setting, implementing, and evaluating care to resolve the problem. A clinical problem other than nursing diagnosis requires a collaborative approach with the appropriate health care professionals.

During a change-of-shift report, a nurse receives information that a patient admitted with hypertensive emergency has prescriptions for antihypertensive medications given at 8 AM and due at 8 PM. During the 8:00 PM assessment, the patient's blood pressure is 90/60, and they report slight dizziness upon standing. After returning the patient to bed, what action will the nurse take? Exhibit: Electronic health record, vital signs 8:00 AM 182/100 12:00 PM 168/98 4:00 PM 160/88 A. Record the BP in the electronic health record B. Notify the health care provider C. Administer the 8:00 PM medications D. Place the patient flat in bed

b. When assessment findings reveal a critical change in the patient's health status, the nurse reports the data (verbally) immediately. The nurse verifies the BP and notifies the health care provider, who may prescribe withholding blood pressure medications, assessment of orthostatic vital signs, among other actions. There is no indication the patient needs to lie flat at this time.

through the gown A nurse is caring for a 25-year-old patient who is unresponsive following a head injury. The patient has several piercings in the ears and nose that appear crusted and slightly inflamed. What is the most appropriate action to care for this patient's piercings? A. Avoiding removing or washing the piercings until the patient is responsive B. Rinsing the sites with warm water and remove crusts with a cotton swab C. Washing the sites with alcohol and apply an antibiotic ointment D. Removing the jewelry and allow the sites to heal over

b. When providing care for piercings, the nurse performs hand hygiene, applies gloves, then cleanses the site of all crusts and debris by rinsing the site with warm water and removing the crusts with a cotton swab. The nurse should then apply a dab of liquid-medicated cleanser, per policy, to the area, turn the jewelry back and forth to work the cleanser around the opening, rinse well, remove gloves, and perform hand hygiene. The nurse should not use alcohol, peroxide, or ointments at the site and should avoid removing piercings unless it is absolutely necessary (e.g., when an MRI is ordered.)

When bathing a patient with C. diff infection, the nurse wears personal protective equipment (PPE). Which additional intervention promotes safe, effective care? A. Donning PPE after entering the patient room B. Bathing the perianal area last C. Personalizing care by substituting glasses for goggles D. Removing PPE after bathing the patient to talk with them in the room

b. When using PPE, the nurse should work from clean areas to soiled ones, don PPE before entering the patient room, always use goggles instead of personal glasses, and remove PPE in the doorway or anteroom just before exiting.

During an assessment, the nurse on a neurologic unit finds the patient confused to time and place but able to state their name. How will the nurse best record this in the electronic health record? A. Is more confused than yesterday B. States the year is 1975 and they are at a wedding C. Disoriented to person, time, and place D. Patient's speech is garbled

b. While the patient is confused, it is most important to clearly describe the behavior for comparison to past and future behavior. Citing the actual year and events (orientation to time and place) provides context; "patient confused" is open to misinterpretation. Garbled speech refers to speech that is unclear; the patient may have difficulty with pronunciation or speak slowly, which is not necessarily reflective of confusion.

A postoperative patient who has a large abdominal incision suddenly calls out for help, shouting, "Something is falling out of my incision!" The nurse notes the wound is gaping open with tissue bulging outward. Place the nursing interventions in the order they should be performed, arranged from first to last. A. Notify the health care provider of the situation. B. Cover exposed tissue with sterile towels moistened with sterile 0.9% sodium chloride solution. C. Place the patient in the low Fowler position. D. Document the findings and outcome of interventions. E. Maintain NPO status for return to the OR for repair.

c, b, a. e. d. The correct order of nursing interventions for this postoperative emergency is to place the patient in the low Fowler position (to prevent further damage or protrusion from increased intraabdominal pressure), cover exposed tissue with sterile towels moistened with sterile 0.9% sodium chloride solution (to protect the viscera), and notify the health care provider of the situation (to address the issue, likely with surgery). The patient is kept NPO, as prompt surgical repair will be needed. After the patient has received attention, the nurse documents all assessments and interventions in a timely manner.

During an interaction with a patient diagnosed with epilepsy, a nurse notes that the patient is silent after hearing the plan of care. How does the nurse best respond? Select all that apply. A. Fill the silence with lighter conversation directed at the patient. B. Use the time to perform the care that is needed uninterrupted. C. Discuss the silence with the patient to ascertain its meaning. D. Allow the patient time to think and explore inner thoughts. E. Determine if the patient's culture requires pauses between conversation. F. Arrange for a counselor to help the patient cope with emotional issues.

c, d, e. Appropriate use of silence allows the patient to initiate or to continue speaking; the nurse can reflect on what has been shared while observing the patient without having to concentrate simultaneously on conversation. In due time, the nurse might discuss the meaning of silence with the patient. The nurse considers whether the patient's culture may require longer pauses between verbal communication. Fear of silence sometimes leads to excessive talking by the nurse, displacing focus from the patient. The nurse should not assume silence requires a consult with a counselor.

A nurse is teaching patients of all ages in a hospital setting. Which teaching examples are appropriate for the patient's developmental level? Select all that apply. A. The nurse plans long teaching sessions to discuss diet modifications for an older adult diagnosed with type 2 diabetes. B. The nurse recognizes that a female adolescent diagnosed with anorexia is still dependent on her parents and includes them in all teaching sessions. C. The nurse designs an exercise program for a sedentary older adult male patient based on the activities he prefers. The nurse includes an 8-year-old patient in the teaching plan for managing cystic fibrosis. The nurse demonstrates how to use an inhaler to an 11-year-old male patient and includes his mother in the session to reinforce the teaching. The nurse continues a teaching session on STIs for a sexually active male adolescent despite his protest that "I've heard enough alr

c, d, e. Successful teaching plans for older adults incorporate extra time, short teaching sessions, accommodation for sensory deficits, and reduction of environmental distractions. Older adults also benefit from instruction that relates new information to familiar activities or information. School-aged children are capable of logical reasoning and should be included in the teaching-learning process whenever possible; they are also open to new learning experiences but need learning to be reinforced by either a parent or health care provider as they become more involved with their friends and school activities. Teaching strategies designed for an adolescent patient should recognize the adolescent's need for independence, as well as the need to establish a trusting relationship that demonstrates respect for the adolescent's opinions.

A nurse performs nurse-initiated nursing actions when caring for patients in a skilled nursing facility. Which reflect these types of actions? Select all that apply. A. Administering an antibiotic to a patient with pneumonia B. Consulting with a psychiatrist for a patient who misuses opiates C. Checking the skin of bedridden patients for skin breakdown D. Ordering a kosher meal for an orthodox Jewish patient E. Recording a patient's intake and output F. Preparing a patient for surgery according to facility protocol

c, d, f. Nurse-initiated interventions, or independent nursing actions, include nurse-prescribed interventions resulting from their assessment of patient's actual or potential health problems. Protocols and standard orders empower the nurse to initiate actions that ordinarily require the order or supervision of a health care provider. Consulting with a psychiatrist is a collaborative intervention.

A nurse is caring for a postoperative patient who has a prescription for morphine 2 mg IV every 3 hours. Which examples documenting pain management best reflect recommended guidelines? Select all that apply. A. 6/12/25 0945 Morphine 2 mg administered IV. Patient's response to pain appears to be exaggerated. M. Patrick, RN. B. 6/12/25 0950 Morphine 2 mg administered IV. Patient appears to be comfortable. M. Patrick, RN. C. 6/12/25 1015 Administered morphine 2 mg IV at 0945, patient reporting pain as 2 on a scale of 1 to 10. M. Patrick, RN. D. 6/12/25 0945 Patient reports severe pain in right lower quadrant. M. Patrick, RN E. 6/12/25 0945 Morphine IV 2 mg will be administered to patient every 3 hours. M. Patrick, RN F. 6/12/25 0945 Patient states they do not want pain medication despite return of pain. After discussion, patient agrees to try morphine 2 mg IV. M. Patrick, RN

c, d, f. The nurse should enter information in a complete, accurate, concise, current, and factual manner, indicating the date and both the time the entry was written and the time of pertinent observations and interventions. When charting, the nurse should avoid the use of stereotypes, derogatory terms, and judgments such as "response to pain appears to be exaggerated" or "seems to be comfortable." Stating that medication will be given does not document care given; this prescription/intervention belongs in the plan of care.

A nurse is administering a medication to a patient via an enteral feeding tube. Which are accurate guidelines related to this procedure? Select all that apply. A. Crushing the enteric-coated pill and mix it in a liquid B. Initially flushing the tube with 60 mL of very warm water C. Using the recommended policy to check tube placement in the stomach or intestine D. Giving each medication separately and flush with water between each drug E. Lowering the head of the bed to prevent reflux F. Adjusting the amount of water used if patient's fluid intake is restricted

c, d, f. The nurse should use the facility policy for checking tube placement prior to administering medications. The nurse should also give each medication separately and flush with water between each drug, adjusting the amount of water used if fluids are restricted. Enteric-coated medications should not be crushed, the tube should be flushed with 15 to 30 mL of water, and the head of the bed should be elevated to prevent reflux and aspiration.

A nurse is assessing a patient's eyes for accommodation. Place the steps of this assessment in order they are performed. A. Document the results in the electronic health record. B. As the patient to look at a distant object, then back to the object held. C. Hold a straight object 0 to 15 cm (4 to 6 inches) from the bridge of the patient's nose. D. Observe for pupillary constriction when looking at the near object and for pupillary dilation when looking at the distant object held. E. Ask the patient to look at the object.

c, e, b, d, a. To test accommodation the nurse holds the forefinger, a pencil, or other straight object about 10 to 15 cm (4 to 6 inches) from the bridge of the patient's nose. The patient is asked to look at the object, then at a distant object, then back to the object being held. The pupil normally constricts when looking at a near object and dilates when looking at a distant object. The patient must be cooperative to complete this assessment.

Nursing students are learning about the criteria that establish nursing as a profession. Which statements by the students indicate they have correctly described characteristics of a profession? Select all that apply. A. Nursing is composed of a well-defined body of general knowledge. B. Nursing interventions are dependent upon medical practice. C. Nursing is a recognized authority by a professional group. D. Nursing practice is regulated by the medical profession. E. Nursing has a code of ethics. F. Nursing is influenced by ongoing research.

c, e, f. Nursing is recognized increasingly as a profession based on the following defining criteria: well-defined body of specific and unique knowledge, strong service orientation, recognized authority by a professional group, code of ethics, professional organization that sets standards, ongoing research, and autonomy and self-regulation.

A nurse caring for patients in a long-term care facility develops strategies to help patients achieve Maslow's highest level of needs: self-actualization. Which concepts will the nurse incorporate when planning care? Select all that apply. A. Humans are born with a fully developed sense of self-actualization. B. Self-actualization needs are met by depending on family, friends, and others for help. C. No matter the patient's age, the self-actualization process continues throughout life. D. Loneliness and isolation occur when self-actualization needs are unmet. E. A person achieves self-actualization by focusing on problems outside self. F. Self-actualization needs may be met by creatively solving problems.

c, e, f. Self-actualization, or reaching your full potential, is a process that continues throughout life. A person achieves self-actualization by focusing on their unique capabilities, being creative, and demonstrating the capacity for happiness and affection for others. Humans are not born with a fully developed sense of self-actualization, and self-actualization needs are not met specifically by depending on others for help. Loneliness and isolation are not always the result of unmet self-actualization needs

A nurse assists a patient with ambulation for the first time following a knee replacement. Shortly after beginning to walk, the patient tells the nurse that they are dizzy and feel like they might fall. Place these nursing actions in the order in which the nurse should perform them to protect the patient: A. Grasp the gait belt. B. Stay with the patient and call for help. C. Place feet wide apart with one foot in front. D. Gently slide the patient down to the floor, protecting their head. E. Pull the weight of the patient backward against your body. F. Rock your pelvis out on the side of the patient.

c, f, a, e, d, b. When a patient is being moved or ambulated and starts to fall, the nurse places their feet wide apart with one foot in front, rocks their pelvis out toward the side of the patient, grasps the gait belt, supports the patient by pulling the patient's weight backward against their body, gently slides the patient down their body toward the floor while protecting the patient's head, and remains with the patient while calling for help.

The nurse preceptor is supervising a new graduate nurse as they assess a patient with a pressure injury. The graduate nurse documents the presence of biofilm in the wound. The preceptor recognizes the graduate nurse understands this concept when the graduate makes which of these statements? Select all that apply. A. Enhanced healing occurs due to the presence of sugars and proteins. B. Delayed healing develops due to dead tissue present in the wound. C. Antibiotics against the bacteria become less effective. D. Skin loses its integrity due to overhydration of the cells of the wound. E. Delayed healing due to cells dehydrating and dying occurs. F. Decreased effectiveness of the patient's normal immune process results.

c, f. Wound biofilms are the result of wound bacteria growing in clumps, embedded in a thick, self-made, protective, slimy barrier of sugars and proteins. This barrier contributes to decreased effectiveness of antibiotics against the bacteria (antibiotic resistance) and decreases the effectiveness of the normal immune response by the patient. Necrosis (dead tissue) in the wound delays healing. Maceration or overhydration of cells related to urinary and fecal incontinence can lead to impaired skin integrity. Desiccation is the process of drying up, in which cells dehydrate and die in a dry environment.

A nursing student is presenting their concept map care plan for a patient with sickle cell anemia in post-conference. How does the student best describe the "concepts" that are being diagrammed in the plan? A. Protocols for treating the patient's medical problem B. Evidence-based treatment guidelines C. Synthesis of the patient's problems and treatment D. Clinical pathways reflecting evidence-based treatment for sickle cell anemia

c. A concept map care plan is a diagram synthesizing patient problems and interventions. The nurse's ideas about patient problems and treatments are the "concepts" that are diagrammed. These maps are used to organize patient data, analyze relationships in the data, and provide a holistic view of the patient's situation. Answers (a) and (b) are incomplete because the concepts being diagrammed may include protocols and standardized treatment guidelines but the patient problems are also diagrammed concepts. Clinical pathways are tools used in case management to communicate the standardized, interdisciplinary care plan for patients.

A nurse working in a new community performs an assessment to determine the health of the community. What finding indicates a healthy community? A. Meets all the needs of its inhabitants B. Mixes residential and industrial areas C. Offers access to health care services D. Consists of modern housing and condominiums

c. A healthy community offers access to health care services to treat illness and to promote health. A healthy community cannot usually meet all the needs of its residents, but should be able to help with health issues such as nutrition, education, recreation, safety, and zoning regulations to separate residential sections from industrial areas. The age of housing is irrelevant as long as residences are maintained according to code.

A patient admitted through the emergency department for a severe infection is receiving intravenous (IV) antibiotics. The patient, who has been oriented, demands the nurse remove the IV because the patient is leaving now. What action will the nurse take? A. Apply soft wrist restraints B. Perform a neurologic assessment C. Explain that after signing an "against medical order form," the patient may leave D. Call the patient's family to encourage the patient to stay

c. A person cannot be legally forced to remain in a health facility, such as a hospital, if that person is of sound mind. The patient signs an "against medical orders" form when insisting on being discharged, to indicate not holding the facility responsible for harm from leaving. Applying soft wrist restraints when the patient has expressed wanting to leave constitutes battery, which includes willful, angry, and violent or negligent touching of another person's body or clothes or anything attached to or held by that other person. The patient has been oriented, so another assessment is not indicated. The patient, not the family, has autonomy.

A nurse is changing a patient's bed linens after drainage from an infected abdominal wound leaked. Which nursing action reflects proper use of medical asepsis? A. Carrying soiled bed linens close to the body to prevent spreading microorganisms into the air B. Placing soiled bed linens and hospital gowns on the floor when making the bed C. Moving the patient table away from the body when wiping it off D. Cleaning the most soiled items at the bedside first, followed by cleaner items

c. According to the principles of medical asepsis, the nurse should move equipment and soiled items away from the body to prevent contaminated particles from settling on the hair, face, or uniform. The nurse should not put soiled items on the floor, as it is highly contaminated. The nurse should also clean the least soiled areas first, then move to the more soiled ones to prevent contamination.

A nurse is writing nursing outcomes in the affective domain for a patient who is trying to stop smoking. Which outcome statement will the nurse include in the care plan? A. "The patient will state the relationship between smoking and coronary artery disease." B. "After the teaching session, the patient will redemonstrate the proper application of a nicotine patch." C. "The patient will state they value a healthy body sufficiently to stop smoking prior to discharge." D. "The patient will state that any changes in cough should be reported to the health care provider"

c. Affective outcomes pertain to changes in patient values, beliefs, and attitudes. Cognitive outcomes involve increases in patient knowledge; psychomotor outcomes describe the patient's achievement of new skills; physical changes are actual bodily changes in the patient (e.g., weight loss, increased muscle tone).

A nurse has performed an admission assessment on a patient. What step does the nurse perform after clustering the data? A. Developing interventions B. Nursing judgments C. Diagnosing and analyzing D. Concept mapping

c. After clustering the data, the nurse analyzes the data and formulates a nursing diagnosis. Interventions are based on and developed after goal setting. Nursing judgments are outcomes based on critical thinking and clinical reasoning. A concept map is a diagram or pictorial representation of the (student) nurse's understanding of the interactions and relationships of the patient's problems and plan of care.

When implementing a thoughtful, patient-centered care plan, which action does the nurse prioritize? A. The patient's loved ones are considered part of the team. B. A caring relationship with mutual trust is established. C. Measures for safety are visibly incorporated. D. Transparent communication is observed.

c. Although developing a thoughtful, patient-centered approach is focused on caring and mutual trust, the nurse uses the nursing process and Maslow's hierarchy of needs to prioritize care. Safety is a higher-level need than love and belonging, and therefore the priority.

A nurse is interviewing a newly admitted patient from another culture. What question best displays cultural sensitivity? A. "Do you think you'll be able to eat the food we have here?" B. "You do understand that we can't prepare special meals?" C. "What types of food do you typically prepare for meals?" D. "Could you make an exception on what food you eat while you are here?"

c. Asking patients what types of foods they eat for meals is culturally sensitive. The other questions are culturally insensitive.

A nursing student is nervous and concerned about working at a clinical facility. Which action would best decrease anxiety and help ensure successful delivery of patient care? A. Determining the established goals of the institution B. Ensuring that verbal and nonverbal communication is congruent C. Engaging in self-talk to plan the day and decrease fear D. Speaking with fellow colleagues about how they feel

c. By engaging in positive self-talk, or intrapersonal communication, the nursing student can plan the day, decrease fear and anxiety, and enhance clinical performance.

A new graduate nurse tells the preceptor they want to obtain recognition in wound care, a specialty area of nursing. What credential will this nurse need to seek? A. Accreditation B. Licensure C. Certification D. Board approval

c. Certification is the process by which a person who has met certain criteria established by a nongovernmental association is granted recognition in a specified practice area. Nursing is one of the groups operating under state laws that promote the general welfare by determining minimum standards of education through accreditation of schools of nursing. Licensure is a legal document that permits a person to offer to the public skills and knowledge in a particular jurisdiction, where such practice would otherwise be unlawful without a license. State board of approval ensures that nurses have received the proper training to practice nursing

A nurse in the emergency department is assessing a young adult who has cognitive disability and is reporting severe abdominal pain. The patient is accompanied by the director of the group home where they live. When collecting data from this patient, which action reflects best practice? A. Ask the assessment questions of the director. B. Wait for the young adult's parents to arrive before performing the assessment. C. Ask the young adult questions and validate with the adult present. D. Perform the physical assessment, then the intake interview when the family arrives.

c. Children and people with decreased mental capacity or impaired verbal ability should be encouraged to respond to interview questions as best as they can. This communicates support of the patient's autonomy, expression of their needs, and respect for their abilities. The information is then validated with family members or guardians as appropriate.

The nurse on a medical-surgical unit attends a class on the seven crucial conversations in health care. After observing a colleague administer an incorrect dose of medication without reporting it, which action will the nurse take? A. Speak to the nurse privately and tell her if she does not complete an event report, you will report her to the unit manager B. Tell the nurse you overheard her discussing giving too much medication, and she must complete an event report or you will C. Explain that you are aware of the medication incident, and you can assist her in notifying the health care provider for patient safety D. Give the nurse a copy of the handout from the class and explain that this class in crucial conversations was very helpful

c. Crucial Conversations for Healthcare, addresses "undiscussable" communication breakdowns and gaps that can result in patient harm, medical errors, and staff turnover. When nurses feel unsafe to report problems or are not heard, dangerous shortcuts, incompetence, and disrespect may ensue. Threatening the nurse with reporting them is unprofessional and inconsistent with the crucial conversations. Providing a handout is passive aggressive and does not clearly relate the medication error to this concept.

A nurse conducting quantitative research hypothesizes that adolescents with anorexia nervosa who participate in outpatient therapy report less depression than those receiving inpatient treatment. What information does the nurse collect to support the hypothesis? A. Subjects' demographics B. Variables C. Data D. Instruments

c. Data refer to information that the researcher collects from subjects in the study, generally expressed in words, numbers, graphs, and charts. A variable is something (such as conditions, equipment, interventions) that varies and has different values (outcomes) that can be measured. Instruments are devices used to collect and record the data, such as rating scales, pencil-and-paper tests, and biologic measurements.

A nurse in the pediatric unit of an acute care hospital is awaiting a prescription for antibiotics for a toddler with a severe infection. Which information about the child is essential to document immediately? A. Beverage preference B. Whether a parent/guardian is present at the bedside C. Weight in kilograms D. Intake and output

c. Drug doses for children are calculated by weight in kilograms or BSA. The weight is needed for the provider to write the prescription.

A group of nurses in the postanesthesia care unit propose a study to determine which of two antiemetic medications from the same classification best relieve postoperative nausea. Before implementing the study, whose approval will the nurses need? A. Organization's surgical director B. Pharmacy director and staff C. Institutional review board D. Director of nursing

c. Federal regulations require that institutions receiving federal funding or conducting studies of drugs or medical devices regulated by the Food and Drug Administration establish institutional review boards (IRBs). The IRBs reviews studies conducted in the institution to determine the risk status of the studies and to ensure that ethical principles are followed. While organizations may have internal procedures for conducting research, such as including surgeons, the director of nursing, the pharmacy director, and staff, these are not government mandates. The type of study, such as randomized-controlled study, will be determined by the nurse in collaboration with the prescriber and approval of the IRB.

A nurse in a pediatric practice teaches the mother of a toddler to administer antibiotic ear drops. What education by the nurse is correct? A. "It is best to pull the pinna of the affected ear up and back." B. "Place the child on their side of the affected ear and pull the pinna straight back." C. "When administering the ear drop, gently pull the pinna down and back." D. "Cleanse the ear canal with a cotton swab before administering."

c. For children ages 3 years and younger, the nurse teaches the parent or caregiver to pull the pinna down and back to straighten the auditory canal, then instill the medication. For adults, the pinna is pulled up and back; for school-age children, the pinna is pulled straight back. The nurse does not insert cotton swabs in the ear; rather, the nurse can wipe the external ear of drainage, if necessary.

A nurse has volunteered to give influenza immunizations at a local health clinic. In which level of health promotion and illness prevention is the nurse participating? A. Tertiary B. Secondary C. Primary D. Promotive

c. Giving influenza injections is an example of primary health promotion and illness prevention. Secondary prevention includes early detection of disease with prompt diagnosis and treatment. Tertiary prevention focuses on reducing disability and providing rehabilitation to a maximum level of functioning

A nurse on a medical-surgical unit notes a patient with pneumonia and is experiencing dyspnea. What action will the nurse take to improve the dyspnea? A. Encourage the patient to ambulate. B. Suggest the patient use music or television as distraction. C. Place the patient in Fowler's position. D. Tell the patient to take several deep breaths, then hold their breath for 5 seconds.

c. High-Fowler's position promotes maximal lung expansion and is the position of choice during episodes of dyspnea. Encouraging ambulation during distress will increase dyspnea. Distracting the patient is not addressing the underlying cause of dyspnea, which is activity. Holding the breath increases demands on the heart.

A nurse is assisting a patient who is 2 days postoperative from a cesarean section dangle in preparation for sitting in a chair. After assisting the patient to stand up, the patient's knees buckle and she tells the nurse she feels faint. What is the appropriate nursing action? A. Supporting the patient as she stands, waiting a few moments, then continuing the move to the chair B. Calling for assistance and continuing the move with the assistance of another nurse C. Lowering the patient back to the side of the bed and pivoting her back into bed D. Having the patient sit down on the bed and dangle her feet before moving

c. If a patient becomes faint and their knees buckle when moving from bed to a chair or ambulating, the nurse should stop the activity, as the patient has demonstrated a clear risk for falling. The nurse should lower the patient back to the side of the bed, pivot her back into bed, cover her, and raise the side rails. Assess the patient's vital signs and for the presence of other symptoms. When vital signs are stable, another attempt can be made with the assistance of another staff. Instruct the patient to remain in the sitting position on the side of the bed for several minutes to allow the circulatory system to adjust to a change in position and prevent hypotension related to a sudden change from the supine position.

A nurse and health care provider are preparing for insertion of a central venous catheter when the patient accidentally touches the sterile field. What action will the nurse take next? A. Ask another nurse to hold the patient's hand and continue setting up the field B. Remove any objects the patient touched and resume setting up the sterile field C. Have someone hold the patient's hand, discard the supplies, and prepare a new sterile field D. No action since the patient has touched their own sterile field

c. If a patient touches a sterile field, the nurse should discard all supplies and prepare a new sterile field. If the patient is restless or confused, the nurse obtains an assistant to hold the patient's hands and explain what is happening.

A nurse administers a dose of an oral medication for hypertension to a patient who immediately vomits after swallowing the pill. What action will the nurse take first? A. Readminister the medication and notify the health care provider. B. Obtain the pill in a liquid form for administration. C. Assess the emesis, looking for the pill. D. Notify the primary care provider.

c. If a patient vomits immediately after swallowing an oral pill, the nurse should assess the emesis (vomit) for the pill or fragments of it. The nurse will collaborate with the health care provider to determine if another dose should be administered.

A community organization includes provision of culturally competent care in their mission. Which action has the organization set as a priority? A. Learning the predominant language of the community B. Obtaining significant information about the community C. Treating each patient at the clinic as an individual D. Recognizing the importance of the patient's family

c. In all aspects of nursing, patients should be treated as individuals; this remains true when providing culturally competent care. Additional ways to provide culturally competent care include learning the predominant language in the community, researching the patient's culture, and recognizing the influence of family on the patient's life.

A health care provider has been urgently paged to another unit and asks a nurse to enter a pain medication prescription for their patient in the electronic medical record. Which response by the nurse is most appropriate? A. "Thank you for taking care of this; I'll be happy to enter a verbal order into the electronic health record." B. Get a second nurse to listen to the order, write the order on the health care provider order sheet, and ensure both nurses sign it. C. "I'm sorry; verbal orders can only be accepted in an emergency. Please enter this quickly before leaving this unit." D. Try calling another resident for the order or wait until the next shift.

c. In most facilities, health care providers may only issue verbal orders in an emergency. The provider is present but finds it impossible, due to the situation, to write or enter the order in the electronic health record. Calling another health care provider or waiting until the next shift would be inappropriate; the patient should not have to wait for the pain medication, and a provider is available who can quickly write/enter the order.

A patient has come to the emergency department with symptoms of a stroke. During the assessment, the nurse asks the patient to raise their eyebrows, smile, and show their teeth to evaluate which cranial nerve? A. Olfactory B. Optic C. Facial D. Vagus

c. Motor function of the facial nerve (cranial nerve VII) is assessed by asking the patient to raise their eyebrow, smile, and show their teeth. The olfactory nerve (cranial nerve I) is tested by testing the sense of smell using various familiar substances. The nurse tests the optic nerve (cranial nerve II) for acuity and visual fields and the vagus nerve (cranial nerve X) by asking the patient to swallow and speak, noting hoarseness.

A nurse is providing education to a patient and their family regarding the use of negative pressure wound therapy (NPWT). The nurse documents that the teaching has been effective when the patient and family make which statement? A. "This therapy is used to collect excess blood loss and prevent formation of a scab." B. "The suction created will prevent infection and promote wound healing with less scar tissue." C. "Suction stimulates blood flow to the wound, removes excess fluid, and promotes a moist environment for healing." D. "This treatment irrigates the wound, suctions the irrigation fluid from the wound, and keeps it free from debris wound exudate."

c. Negative pressure wound therapy (NPWT) promotes wound healing and wound closure through the application of uniform negative pressure on the wound bed, reduction in bacteria in the wound, and the removal of excess wound fluid, while providing a moist wound healing environment. The negative pressure results in mechanical tension on the wound tissues, stimulating cell proliferation, blood flow to wounds, and the growth of new blood vessels. It is used to treat a variety of acute or chronic wounds, wounds with heavy drainage, wounds failing to heal, or healing slowly.

A nursing professor pulls a student aside to discuss documenting a patient's blood pressure of 202/122 but not reporting this to the primary nurse. When discovered, the patient was transferred to the intensive care unit for treatment and monitoring. How does the faculty best explain to the student that their inaction reflects negligence? A. "You did not re-assess your patient." B. "There was poor interprofessional communication with the health care team." C. "You failed to act as a reasonably prudent nurse would under similar circumstances." D. "This action is consistent with a felony criminal action."

c. Negligence is defined as performing an action that the reasonably prudent nurse would not perform or failing to act as a reasonably prudent nurse would in similar circumstances. Negligence may be an act of omission or commission. Criminal law concerning state and federal criminal statutes includes murder, manslaughter, criminal negligence, theft, and illegal possession of drugs. Public law regulates relationships between people and the government. Private or civil law includes laws relating to contracts, ownership of property, and the practice of nursing, medicine, pharmacy, and dentistry.

A school nurse assesses adolescents' visual acuity using a Snellen eye chart. Which explanation does the nurse provide to the student whose vision is 20/40 in both eyes? A. "They see better than 50% of people." B. "Double vision is present." C. "Vision is less than normal." D. "They have normal vision."

c. Normal vision is 20/20. The higher the denominator indicates increasingly worse vision; 20/40 vision indicates less than normal vision.

A nurse is administering medications to an older adult with dysphagia. After crushing the pills, which action is most appropriate? A. Mixing the crushed medications with 120 mL of water before administering B. Mixing the medications into the patient's bowl of pudding C. Crushing each pill separately and administering each in a teaspoon of applesauce D. Asking the patient to chew the pills and providing juice after swallowing

c. Older adults may have difficulty swallowing medications and may find it easier to take their medications when crushed or given in liquid form. Combine the crushed medication with a small amount of soft food, such as applesauce or pudding; 120 mL or a bowl of pudding is a large amount; should the patient not finish all the food, an accurate assessment of medication taken cannot be assessed. Only medications designated as chewable can be used in this way.

Nurses on a hospital unit work to improve staff communication, as outlined in The Joint Commission's National Patient Safety Goals. What process will best provide for continuity of the plan of care? A. Checking two patient identifiers, such as name and date of birth, prior to administering medications B. Ensuring two nurses check doses of high-risk medications such as anticoagulants or insulin C. Giving handoff report in the patients' rooms to update the next nurse on the plan of care D. Obtain a patient sitter for a confused individual who has fallen trying to get out of bed

c. One of the published standards and requirements for accreditation and certification required by The Joint Commission is to "improve staff communication." Communicating the plan of care with the patient and oncoming and off going nurses meets this goal. Using patient identifiers relates to the goal of safely identifying patients, checking high-risk medications relates to decreasing medication errors, and obtaining a patient sitter relates to general safety and fall reduction.

A nurse is conducting quantitative research to examine which of two types of silicone foam sacral dressings best prevent pressure injuries in bedridden patients. What type of research is the nurse conducting? A. Descriptive B. Correlational C. Quasi-experimental D. Experimental

c. Quasi-experimental research is often conducted in clinical settings to examine the effects of nursing interventions on patient outcomes. Descriptive research is often used to generate new knowledge about topics with little or no prior research. Correlational research examines the type and degree of relationships between two or more variables. Experimental research examines cause-and-effect relationships between variables under highly controlled conditions.

A patient who is receiving cancer chemotherapy tells the nurse, "The treatment for this cancer is worse than the disease itself. I'm stopping treatment." Which nursing action best promotes a patient-centered, therapeutic relationship? A. Determining if the patient database is adequate to address the problem B. Considering whether to suggest a counseling session for the patient C. Reassessing the patient and determining how to best support them D. Identifying possible interventions and critiquing the merit of each option

c. Reassessing the patient allows the nurse and patient to clarify the patient's goal(s) and develop interventions to best meet them. Once the problem is addressed, it is important for the nurse to judge the adequacy of the knowledge, identify potential problems, use helpful resources, and critique the decision.

A home care nurse is assisting an older adult with an unsteady gait with a tub bath. Which action is recommended in this procedure? A. Adding bath oil to the water to prevent dry skin B. Allowing the patient to lock the door to guarantee privacy C. Assisting the patient in and out of the tub to prevent falling D. Keeping the water temperature very warm because older adults chill easily

c. Safe nursing practice requires that the nurse assists a patient with an unsteady gait in and out of the tub. Adding bath oil to the bath water poses a safety risk because it makes the patient and tub slippery. Although privacy is important, if the patient locks the door, the nurse cannot help if there is an emergency. The water should be comfortably warm at 43° to 46°C. Older adults have an increased susceptibility to burns due to diminished sensitivity.

A nurse is teaching an adult patient how to care for their new ostomy appliance. Which evaluation method is most appropriate to confirm that the patient has learned the information? A. Ask Me 3 B. Newest Vital Sign (NVS) C. Teach-Back Method D. TEACH acronym

c. The Teach-Back Method tool is a method of assessing literacy and confirming that the learner understands health information received from a health professional. The Ask Me 3 is a brief tool intended to promote understanding and improve communication between patients and their providers. The NVS is a reliable screening tool to assess low health literacy, developed to improve communications between patients and providers. The TEACH acronym is used to maximize the effectiveness of patient teaching by tuning into the patient, editing patient information, acting on every teaching moment, clarifying often, and honoring the patient as a partner in the process.

A nurse is explaining to a patient the anticipated effect of the application of cold to an injured area. What response indicates that the patient understands the explanation? A. "There will be more discomfort in the area where the cold is applied." B. "I should expect more drainage from the incision after the ice has been in place." C. "Redness and swelling should decrease after cold treatment." D. "My incision may bleed more when the ice is first applied."

c. The local application of cold constricts peripheral blood vessels, reduces muscle spasms, and promotes comfort. Cold reduces blood flow to tissues, decreases the local release of pain-producing substances, decreases metabolic needs, and capillary permeability. The resulting effects include decreased edema, coagulation of blood at the wound site, promotion of comfort, decreased drainage from wound, and decreased bleeding.

A patient with dysphagia has been admitted with a third episode of aspiration pneumonia in less than a year. The health care provider insists that for safety, the patient must have a feeding tube placed. The patient declines the tube and later asks the nurse if they should reconsider. What is the most appropriate nursing response? A. "The feeding tube will prevent aspiration and is the safest option for you." B. "You could swallow thickened liquids and puréed foods more easily." C. "Tell me your understanding of what may happen without the feeding tube." D. "You said cooking and dining with your family is important, and I understand your decision."

c. The nurse assesses the patient's understanding of the cause and consequences of aspiration and pneumonia. The nurse can provide further information on pneumonia and sepsis if needed, while supporting the patient's decision. The patient has already declined treatment; therefore, the nurse assesses whether the patient understands the decision before discussing the intervention. The nurse may suggest safer swallowing options or consultation with a speech therapist after assessing the patient's knowledge.

A nurse is caring for a patient recovering from a stroke that paralyzed the dominant arm. The nursing assistant reports that the patient was unable to bathe, comb their hair, or brush their teeth. Which health problem should the nurse add to the care plan? A. Lack of motivation to complete self-care activities B. Risk for: Activities of Daily Living Deficit C. ADL deficit: impaired dressing and grooming D. Impaired musculoskeletal system function: paralysis

c. The nurse clusters the data that demonstrates the patient's (actual) inability to perform bathing and grooming. For that reason, a "potential problem" or "risk" is not appropriate. There is no evidence the patient lacks motivation, and paralysis is not a problem the nurse can resolve.

After receiving a change-of-shift report, the nurse on a medical-surgical unit sets initial priorities for care. According to Maslow's hierarchy of needs, which patient requires immediate assessment? A. Patient requesting help to phone family to ask them to visit B. Patient who needs education on changing their wound dressing prior to discharge C. Patient who calls for assistance because they are breathing fast and feel faint D. Patient who needs assistance to walk to the bathroom to void

c. The nurse prioritizes the patient with rapid breathing who feels faint, who may need oxygen or additional assistance with physiologic needs. Family visits help meet love and belonging needs, while assisting the patient to prevent falls and proper dressing change technique meets safety and security needs.

A nurse in a rehabilitation facility is preparing to administer a skeletal muscle relaxant to a patient recovering from a motor vehicle accident. When the patient states, "I don't want that pill," what action will the nurse take next? A. Encourage the patient to take the pill to help reduce muscle spasm. B. Explain that the health care provider prescribes only necessary medications. C. Ask the patient to explain their concern about the medication. D. Question the patient about allergies and previous medication reactions.

c. The nurse should first assess the patient's concern about the medication and provide education, if needed. If an allergy is the cause, the nurse should withhold the medication and notify the health care provider. Encouraging or manipulating a patient to take a pill undermines the patient's trust, especially if an adverse reaction occurs.

A nurse caring for a group of patients uses measures to reduce discomfort for the patients during injections. Which technique is recommended? A. Selecting a needle of the largest gauge that is appropriate for the site and solution to be injected B. Injecting the medication into contracted muscles to reduce pressure and discomfort at the site C. Using the Z-track technique for intramuscular injections to prevent leakage of medication into the needle track D. Applying vigorous pressure in a circular motion after the injection to distribute the medication to the intended site

c. The nurse should use the Z-track technique for intramuscular injections to prevent medication from leaking into the needle track, minimizing discomfort. The nurse should select a needle of the smallest gauge that is appropriate for the site and solution to be injected, and select the correct needle length for the type of injection and patient size. The nurse should inject the medication into relaxed muscles since more pressure (causing discomfort) will be required to injected into contracted muscles. The nurse should apply gentle pressure after injection, unless this technique is contraindicated.

A nurse in a memory care unit is assisting a patient with dementia with bathing. Which nursing action will enhance patient comfort and prevent anxiety? A. Shifting the focus of the interaction to the "process of bathing" B. Washing the face and hair at the beginning of the bath C. Using music to soothe anxiety and agitation D. Avoiding towel baths or forms of bathing with which the patient is unfamiliar

c. The nurse use music to soothe anxiety and agitation. The nurse should also shift the focus of the interaction from the "task of bathing" to the needs and abilities of the patient, and focus on comfort, safety, autonomy, and self-esteem, in addition to cleanliness. Wash the face and hair at the end of the bath or at a separate time. Water dripping in the face and having a wet head are often the most upsetting parts of the bathing process for people with dementia. The nurse should also consider methods for bathing aside from showers and tub baths. Towel baths, washing under clothes, and bathing "body sections" one day at a time, as well as dry shampoo or "shower cap" shampoos, are additional options.

The nurse is formulating a care plan for a patient in a long-term care facility who has lost 12 lb in the last 2 months. To arrive at a patient-centered nursing judgment, what will the nurse do first? A. Ensure the patient is receiving foods they like, including favorites. B. Make sure the patient's dentures are clean and inserted at mealtimes. C. Assess the patient's food intake and hydration over the last 1 to 3 days. D. Request that the nursing assistant feed the client at mealtime.

c. The nurse uses the nursing process to arrive at a clinical judgment. After analyzing the assessment data, the nurse determines, through clinical reasoning, whether the related factors in the patient's weight loss, such as dislike of menu options, lack of dentition, or inability to perform activities of daily living such as feeding, should be the focus of interventions.

The nurse notes a temperature of 102°F in a patient scheduled for surgery in 30 minutes. As the patient has been afebrile and asymptomatic until now, what action will the nurse take next? A. Inform the charge nurse B. Notify the surgeon C. Reassess the temperature D. Document the finding in the electronic health record

c. The nurse validates assessment findings that deviate from normal patterns or are unsupported by other data. Should the initial measurement be in error, it would have been premature to notify the charge nurse or the surgeon. The nurse must be sure that all data are accurate prior to documenting and reporting. If there is a question about accuracy, the data should be validated before documenting.

A nurse discovers that a medication error occurred. What is the nurse's priority? A. Recording the error on the medication sheet B. Notifying the physician regarding course of action C. Assessing the patient for adverse effects of the error D. Completing an event report, explaining how the mistake was made

c. The nurse's first responsibility is the patient—careful observation is necessary to assess for adverse effects of the medication error. The other nursing actions are pertinent but only after evaluating the patient's welfare.

The nurse on an oncology unit is caring for a patient admitted for dyspnea and wheezing. What is most important for the nurse to include in the change-of-shift report to the oncoming nurse? A. Partial bath was given B. Patient received physical therapy C. CT scan revealed a mass in the right lung D. Patient did not eat lunch today

c. The nurse's shift or handoff report includes basic identifying information about the patient. The outgoing nurse includes the patient's current health status and changes during their shift, response to nursing and medical therapy, pertinent monitoring and assessment findings (e.g., lab and radiology data), pain management, changes in orders (medications, intravenous fluids, diet, and rationale), upcoming/ongoing tests and procedures, and instructions for these, such as NPO after midnight, unfilled prescriptions for the next shift to follow up on, and patient and family questions, concerns, and needs

A nursing student is assigned to the emergency department (ED) to shadow the triage nurse. What activity will the student expect to perform? A. Acute and emergency interventions B. Daily care and assistance with ADLs C. Assessment and prioritization of care D. Care planning for return to home

c. The triage nurse screens patients to determine the extent and severity of their problems. They use highly specialized nursing knowledge and clinical reasoning and make clinical judgments to prioritize who must be seen immediately and who can wait. Patients in the ED are stabilized and transferred to the appropriate level of care; therefore, daily care, assistance with ADLs, planning for return home, and providing interventions are not part of the triage nurse's role. Should the patient need emergency interventions, the triage nurse moves the patient to the appropriate area in the ED.

A nurse is about to bathe a female patient who has an IV in the forearm. The patient's gown, which does not have snaps on the sleeves, needs to be removed prior to bathing. How will the nurse proceed? A. Quickly disconnecting the IV tubing closest to the patient and thread it through the gown sleeve B. Cutting the gown with scissors to allow arm movement C. Threading the bag and tubing through the gown sleeve, keeping the line intact D. Temporarily disconnecting the tubing from the IV container, threading it through the gown

c. Threading the bag and tubing through the gown sleeve maintains a closed system and prevents contamination. No matter how quickly performed, any disconnection of IV tubing results in a breach of the sterile system, creating risk for infection. Cutting a gown is not an alternative except in an emergency.

A nurse develops a care plan for an adolescent patient who gave birth to a premature infant. When presented with the collaborative care plan, including home health care visits, the patient states, "We will be fine on our own. I don't need any more care." What is the nurse's best response? A. "You know your personal situation better than I do; I will respect your wishes." B. "If you don't accept these services, your baby's health will suffer." C. "Let's take a look at the plan again and see if we can adjust it to fit your needs." D. "I'm going to assign your case to a social worker who can explain the services better."

c. When a patient rejects the care plan, the nurse works to identify the underlying barriers. If the nurse determines that the care plan is adequate, the nurse works with the patient to formulate mutually developed goals and interventions.

The care plan for a patient just diagnosed with diabetes contains the expected outcome: "the patient will correctly measure the insulin dose and self-administer the injection, using correct technique by 12/12/24." The nurse observes the client fumbled with the syringe and drew up less insulin than prescribed. What action will the nurse take first? A. Document that the plan of care was unsuccessful B. State continuation of the care plan is indicated C. Assess the patient's vision and dexterity and revise the plan D. Designate a family member to administer the insulin

c. When an outcome is not achieved, the nurse can (1) delete or modify a diagnosis; (2) revise the diagnosis, making it more realistic; (3) adjust the time criteria, or (4) modify the nursing interventions. This outcome was not successfully met; further assessment and revision is indicated. It is inappropriate for the nurse to designate a family member to take over insulin administration without additional assessment and patient permission.

During physical assessment, a nurse inspects a patient's abdomen. What assessment technique does the nurse perform next? A. Percussion B. Palpation C. Auscultation D. Whichever provides patient comfort

c. When assessing the abdomen, the sequence for assessment is: inspection, auscultation, percussion, and palpation. Auscultation follows inspection to avoid stimulating bowel sounds during percussion.

A nurse is administering heparin subcutaneously to a patient. What technique is appropriate for this injection? A. Aspirating before giving and gently massage after the injection B. Avoiding aspirating; massaging the site for 1 minute C. Avoiding aspirating before and massaging after the injection D. Massaging the injection site; aspirating is unnecessary but will do no harm

c. When giving heparin subcutaneously, the nurse should not aspirate or massage, so as not to cause trauma or bleeding in the tissues.

A nurse is getting a patient with right hemiparesis out of bed to the chair. What will the nurse say to the patient? A. "Stand on the weaker leg and pivot toward the chair." B. "I will call the lift team to carry you to the chair." C. "The chair is by your non-affected leg for smoother movement." D. "Avoid putting your hospital socks on, as that will restrict your feet moving."

c. When transferring a patient, the chair is placed on the unaffected or stronger side, rather than the weaker or affected side. Lifting and carrying a patient unless absolutely necessary poses an unnecessary risk for injury to patient and staff. Patients should wear proper shoes, sturdy slippers, or hospital-issued socks with grips to prevent sliding and/or falling.

A nurse in the diabetes clinic initiates education for a patient with a new diagnosis of diabetes. The nurse notes the patient has completed 2 years of college. What action does the nurse select for the initial teaching session? A. Providing the patient with handouts related to blood-glucose management B. Demonstrating the use of the blood-glucose monitor and tool to record blood-glucose readings C. Assessing the patient's knowledge of diabetes and their ability to interpret the health information D. Explaining the dietary restrictions including foods that are prohibited

c. While the nurse takes the patient's level of education into account when providing teaching, the first step is assessing the patient's knowledge and readiness to learn. The nurse would not provide handouts or AV material without reviewing the information and assessing the patient's understanding. Discussing or demonstrating psychomotor skills, an intervention, is performed after assessment.

A patient was in an automobile accident and received a wound across the nose and cheek. After surgery to repair the wound, the patient says, "I am so ugly now." Based on this statement, psychosocial problem will the nurse plan to address? A. Pain B. Wound healing C. Body image D. Change in cognition

c. Wounds cause emotional as well as physical stress.

A nurse uses the ISBARR format to report the deteriorating mental status of a patient using morphine via a patient-controlled analgesia pump (PCA) for postoperative pain. Place the following nursing statements related to this call in the correct ISBARR order. A. "I am calling about Mr. Sanchez in Room 202 who is receiving morphine via a PCA pump for pancreatic cancer." B. "Mr. Sanchez has been difficult to arouse, and his mental status has declined over the past 12 hours." C. "You want me to discontinue the PCA pump until you see him tonight at patient rounds." D. "I am Rosa Clark, an RN working on the second floor of South Street Hospital." E. "Mr. Sanchez was admitted 2 days ago following a diagnosis of pancreatic cancer." F. "I suggest a decrease in the dose of morphine."

d, a, e, b, f, c. The order for ISBARR is: Identity/Introduction, Situation, Background, Assessment, Recommendation, and Read-back.

A nurse is caring for patients of diverse cultures in a community health clinic. Which concepts will the nurse incorporate to guide the plan of care? Select all that apply. A. The United States has become less inclusive of same-sex couples. B. Cultural diversity is limited to people of varying cultures and races. C. Cultural diversity is separate and distinct from health and illness. D. People may be members of multiple cultural groups at one time. E. Culture guides what is acceptable behavior for people in a specific group. F. Cultural practices may evolve over time but mainly remain constant.

d, e, f. A person may belong to multiple cultural, ethnic, and racial groups at one time. Culture guides what is acceptable behavior for people in a specific group. Cultural practices and beliefs may evolve over time, mainly remaining constant as long as they satisfy a group's needs. The United States has become more inclusive of same-sex couples. The definition of cultural diversity includes, but is not limited to, people of varying cultures, racial and ethnic origin, religion, language, physical size, biologic sex, sexual orientation, age, disability, socioeconomic status, occupational status, and geographic location. Cultural diversity, an integral component of health and illness, includes culture, ethnicity, and race.

When performing sterile wound irrigation and dressing change for a postoperative patient, a new graduate nurse creates a sterile field. Which actions require correction by the preceptor? Select all that apply. A. Placing the bottle cap for the irrigating solution off the sterile field with the edges down B. Holding the bottle of irrigating solution inside the edge of the sterile field C. Applying the second sterile glove by lifting it from beneath the cuff with the thumb held away from the glove D. Pouring the irrigating solution into a sterile container from a height of 4 to 6 inches (10 to 15 cm) E. Opening packages of sterile gauze dressings, prior to applying sterile gloves

d, e. To add a sterile solution to a sterile field, the nurse opens the solution container according to directions and places the cap on the table away from the field with the edges up. The nurse then holds the bottle outside the edge of the sterile field with the label side facing the palm of the hand and pours from a height of 4 to 6 inches (10 to 15 cm) to prevent splashing.

A nurse in the psychiatric clinic is developing a problem list for a patient. What statement best reflects a correctly written, two-part problem? A. Difficulty Coping: Impaired Family Coping Etiology: inability to maintain marriage B. Difficulty Coping: Impaired Acceptance of Health Status Etiology: anger management issues C. Impaired Cognition: Distorted Thought Process Etiology: psychosis as evidence by hallucinations D. Impaired Cognition: Decisional Conflict Etiology: placement of parent in a long-term care facility

d. A correctly written two-part problem statement includes the health problem and the etiology or cause. The problem statement and etiology should avoid signs and symptoms, medical diagnoses, and something that cannot be changed. Inability to maintain marriage and anger issues do not identify the underlying cause of the problem and may themselves reflect the true problem. Psychosis is a medical diagnosis, which should not be used to support a patient problem.

A nurse is caring for a patient who had abdominal surgery yesterday. The nurse observes the patient guarding the area with hands and a pillow, refusing to move, and grimacing. What information does the nurse use to formulate the health problem statement? A. Symptoms B. Diagnostic statement C. Etiology D. Cue

d. A cue denotes significant data or "red flags," that, when occurring in a pattern or cluster, point to the existence of a health problem.

A nurse considers returning to school to obtain a master's degree in nursing. Which practice discipline requiring graduate level education could the nurse consider? A. Rehabilitation nurse B. Critical care nurse C. Oncology nurse D. Nurse practitioner

d. A master's degree (MSN) prepares advanced practice nurses, nurse practitioners (NPs), nurse midwives, clinical specialists, or clinical nurse leaders.

A nursery nurse notifies the nurse practitioner (NP) that a newborn has signs of jaundice. The NP performs a brief skin assessment, then orders a blood test for bilirubin levels. Which type of assessment has the NP performed? A. Comprehensive B. Initial C. Time-lapsed D. Quick priority

d. A quick priority assessment (QPA) is a short, focused assessment to obtain the most important information first. A comprehensive initial assessment is performed shortly after admission. The time-lapsed assessment is used to compare a patient's current status to baseline data obtained earlier.

A patient being discharged from the hospital asks to receive a copy of their medical record. What information will the nurse give the patient? A. "I'm sorry, but patients are not allowed to copy their medical records." B. "I can make a copy of your record for you right now." C. "You can read your record while you are still a patient, but copying records is not permitted according to HIPAA rules." D. "I'll check with the medical records department to determine how you request a copy."

d. According to HIPAA, patients have a right to view and receive a copy of their health record; update their health record; get a list of the disclosures a health care institution has made, independent of disclosures made for the purposes of treatment, payment, and health care operations; request a restriction on certain uses or disclosures; and choose how to receive health information. The nurse should be aware of facility policies regarding the patient's right to access and copy records.

A patient with brain cancer questions whether they should agree to palliative radiation treatments despite possible memory loss, or enjoy their remaining time with friends and family. What is the most appropriate response by the nurse? A. "I can't advise you. This is such an individual decision." B. "If you receive the radiation, might you live a bit longer?" C. "What does your family think you should do?" D. "What is most important to you with the time you have left?"

d. Advocacy is the protection and support of another's rights. The nurse assesses the patient's goals and advocates to support their wishes. Nurses do not make ethical decisions for patients; rather; the nurse facilitates patient decision making by interpreting and providing information, encouraging verbalization of feelings, and facilitating communication with family, primary nurse, health care provider, or clergy.

A nurse is developing outcomes in the affective domain for a patient with a foot ulcer related to diabetes. Which outcome best addresses this domain? A. Within 1 day after teaching, the patient will list three benefits of continuing to apply moist compresses to foot ulcer after discharge. B. By 6/12/25, the patient will correctly demonstrate application of wet-to-dry dressing on the foot ulcer. C. By 6/19/25, the patient's pressure ulcer will decrease in size from 3 to 2.5 inches. D. By 6/12/25, the patient will verbalize they value their health sufficiently to control diabetes and prevent recurrence of diabetic ulcers.

d. Affective outcomes describe changes in patient values, beliefs, and attitudes. Cognitive outcomes (a) describe increases in patient knowledge or intellectual behaviors; psychomotor outcomes (b) describe the patient's achievement of new skills; and (c) is an outcome describing a physical change in the patient.

A nurse incorporates concepts from current models of health when providing health promotion classes for patients. What is a key concept of both the health-illness continuum and the high-level wellness models? A. Illness as a fixed point in time B. The importance of family C. Wellness as a passive state D. Health as a dynamic state

d. Both these models view health as a dynamic (constantly changing state).

A nurse is developing a problem list for a care plan. Which reflects a correctly written three-part problem statement? Select all that apply.

d. Correctly written problem statements contain a problem the nurse can treat with independent or interdependent interventions, a clearly stated etiology or cause of the problem, and supporting signs and symptoms. Option (a) may be more easily resolved with the problem statement, knowledge deficiency. Option (a) further states the tube feeding is the underlying cause of the problem; it is a factor that cannot be changed. Option (c) is written in terms of needs and not an unhealthy response. Option (e), while written in three parts, places blame or implies negligence, which is legally inadvisable and should be avoided. A clear etiology is not stated in option (e), impeding direction for appropriate interventions or outcomes.

A nurse is caring for a patient who refuses to look at or care for a new colostomy. The patient states, "I don't care what I look like anymore. I'm not washing up, let alone touching or changing this bag!" The nurse formulates the health problem: Difficulty Coping: Impaired Acceptance of Health Status, reflecting which type of health problem? A. Collaborative B. Interdisciplinary C. Medical D. Nursing

d. Difficulty Coping: Impaired Acceptance of Health Status is a nursing problem, falling within the scope of independent nursing practice. Collaborative and interdisciplinary problems are resolved through a teamwork approach with other health care professionals. A medical problem is a traumatic or disease condition validated by medical diagnostic studies.

A nurse is caring for a patient who has been managing their chronic spasticity well for 10 years. The patient states having had increasing spasticity and few falls due to unremitting muscle spasms. The nurse, patient, and health care provider discuss the possibility the patient may be experiencing which phase of illness? A. Acute onset of illness B. Permanent complication C. Need for hospice care D. Exacerbation of disease

d. Exacerbation of a chronic illness occurs when the symptoms of a disease reappear after having been absent for a time

A nurse in a family-centered health clinic is assessing a new family composed of two parents and three preschool children. Which value does the nurse include in a family-centered approach to health care? A. Each person in the family will be evaluated and treated independently of the others. B. Time will be saved as there is only one clinic to contact for health problems. C. All members of the family can be part of health-related decisions. D. Interdependence of family members affects them in illness and health.

d. Families likely share beliefs and values about health and illness. The nurse can help reduce risk for all family members at any level of development and recognizes interdependence of families affect one another in illness and health promotion.

A nurse is performing oral care on a patient who has advanced dementia. The nurse notes that the mouth is extremely dry with crusts remaining after the oral care. What action will the nurse take next? A. Recommend a consultation with an oral surgeon. B. Communicate the condition to the health care team. C. Gently scrape the oral cavity with a tongue depressor. D. Increase the frequency of the oral hygiene and apply mouth moisturizer to oral mucosa.

d. If initial oral care results in continued dryness of the oral cavity with crusting, the nurse should increase frequency of oral hygiene, apply mouth moisturizer to oral mucosa, and monitor fluid intake and output to ensure adequate intake of fluid. It is not necessary to report this condition prior to providing the interventions mentioned above; however, mouth care and re-evaluation of the oral cavity is documented. The crusts should not be scraped with a tongue depressor.

A nurse working in a long-term care facility reviews the electronic health records of patients who have fallen in the last month to determine if there is a common risk factor. Which QSEN competency is the nurse demonstrating? A. Patient-centered care B. Evidence-based practice C. Teamwork and collaboration D. Informatics

d. Informatics uses information and technology to communicate, manage knowledge, mitigate error, and support decision making. Thoughtful, patient-centered care emphasizes recognition of the patient or designee as the source of control and full partner in compassionate and coordinated care, based on respect for patients' preferences, values, and needs. Evidence-based practice integrates the best current evidence with clinical expertise and patient and family preferences and values to deliver optimal health care. Teamwork and collaboration refer to effective functioning within nursing and interprofessional teams, fostering open communication, mutual respect, and shared decision making to achieve quality patient care.

A patient died during routine outpatient surgery, and the nurse was accused of having failed to monitor and interpret vital signs. Which fact must be established to prove them guilty of malpractice or negligence? A. The surgeon testifies the nurse's action was pure negligence, saying that the patient could have been saved. B. This patient should not have died since they were healthy, physically active, and involved in the community. C. The nurse intended to harm the patient and was willfully negligent, as evidenced by the tragic outcome. D. The nurse had a duty to monitor the patient, and due to the nurse's failure to perform this duty, the patient died.

d. Liability involves four elements that must be established to prove that malpractice or negligence has occurred: duty, breach of duty, causation, and damages. Duty refers to an obligation to use due care (what a reasonably prudent nurse would do) and is defined by the standard of care appropriate for the nurse-patient relationship. Breach of duty is the failure to meet the standard of care. Causation, the most difficult element of liability to prove, shows that the failure to meet the standard of care (breach) caused the injury. Damages are the actual harm or injury resulting to the patient.

A medication prescription reads: "Hydromorphone, 2 mg IV every 3 to 4 hours PRN pain." The prefilled cartridge is available with a label reading "Hydromorphone 2 mg/1 mL" and a statement that the cartridge contains 1.2 mL of hydromorphone. How should the nurse proceed? A. Give the entire contents of the cartridge sent by the pharmacy B. Call the pharmacy and request the proper dose C. Refuse to give the medication and document refusal in the EHR D. Discard 0.2 mL before administration; verify the waste with another nurse

d. Many prefilled medication cartridges are overfilled, requiring some medication to be discarded to prevent adverse effects. Always check correct dose compared to the volume in the syringe for accuracy. As this is a prefilled syringe, it is not necessary to call the pharmacy or refuse to give the medication. Wasting narcotics typically requires a second RN to witness the waste and verify the amount of narcotic discarded.

A nurse follows a prescription written by the health care provider to administer a medication to which the patient is allergic. How does the nurse interpret their liability for administering this medication? A. The nurse is not responsible because they were following the provider's orders. B. The nurse is responsible because they administered the medication. C. The health care provider is responsible because they ordered the drug. D. The nurse, health care provider, and pharmacist bear responsibility for their actions.

d. Nurses are legally responsible for carrying out the orders of the health care provider in charge of a patient unless an order would lead a reasonable person to anticipate injury if it was carried out. If the nurse should have anticipated injury and did not, both the prescribing health care provider and the administering nurse are responsible for the harms to which they contributed.

The nurse manager reviews the medical record of a patient who has accused a nurse of negligence after requiring a "needless" admission to the intensive care unit postoperatively. Which entry in the electronic health record requires follow-up by the manager? Exhibit: Electronic health record (EHR) Nursing Notes: Postoperative follow-up 12:20 pm: patient still reporting incisional pain of 10/10, provider contacted, increased morphine from 1 mg to 2 mg every hour 2: 15 pm: dime-sized, dark red-brown blood stain on dressing; area circled 2:30 pm: patient reports incisional pain, 7/10, 2 mg morphine administered 2:45 pm: vital signs T 99.2°, P 120, RR 20, BP 84/48; will continue to monitor A. Inappropriately recorded vital signs B. Pain treated without appropriate assessment C. Failure to follow up on tachycardia and hypotension D. Lack of interpretation of vital signs and follow-up

d. Nurses are responsible for gathering assessment data including vital signs and interpreting them considering the patient's condition and trends. The nurse did not document interventions from the health care provider for typical symptoms of shock, including tachycardia and hypotension.

After their clinical experience, nursing students are given a reflective assignment to discuss the concepts in nursing theory that influence and determine nursing practice. What part of this theory is most important when delivering thoughtful care? A. Environment B. Health C. Nursing D. Person

d. Of the four concepts, the most important is the person. The focus of nursing, regardless of definition or theory, is the person.

A nurse receives a prescription for an analgesic for a patient who has compound fractures of the tibia and fibula. What schedule will the nurse use to administer the medication?(EHR) Health care provider order sheet 8:00 AM: Hydromorphone 1 mg IV every 2 hours PRN severe pain. -S. Jones, MD A. When the patient requests it B. Every 2 hours on the even hours C. Daily, every 2 hours D. As requested, 2 hours or more after the last dose

d. PRN means "as needed." The nurse teaches the patient that the medication may be requested every 2 hours to treat pain. If pain occurs before 2 hours elapses, the nurse provides comfort measures and collaborates with the health care provider for a change in prescription.

During shift report, a nurse says that a patient has no integumentary changes or skin care needs. During assessment, the nurse observes reddened areas over bony prominences. What action will the nurse take? A. Correct the initial assessment form B. Redo the initial assessment and document current findings C. Conduct and document an emergency assessment D. Perform and document a focused assessment of skin integrity

d. Perform a focused skin assessment for the new problem, documenting the current date. The initial assessment was entered in the permanent health record, correct at the time, and cannot legally be rewritten. An emergency assessment is performed for a life-threatening problem.

A patient states, "I have been experiencing complications of diabetes." What question will the nurse use to elicit additional information? A. "Do you take two injections of insulin to prevent complications?" B. "Are you using diet and exercise to help regulate your blood sugar?" C. "Have you been experiencing the complications of neuropathy?" D. "Can you tell me about the complications you've experienced?"

d. Requesting information regarding the patient's specific complications of diabetes will guide the nurse to further questioning and related assessments.

A university student works with the student health nurse to develop a weight loss plan that includes increasing activity and avoiding empty calories. At the next session, the student has lost 1 lb instead of the projected 5 lb. What action will the nurse take next? A. Congratulate the student and continue the care plan B. Terminate the care plan since it is not working C. Give the student more time to reach the targeted outcome D. Modify the plan after discussing possible reasons for partial success

d. Since the student has only partially met her outcome, the nurse should first explore the factors making it difficult for her to reach her outcome and then modify the care plan. As the plan is not completely working as written, continuing without further assessment is contraindicated. It is premature to terminate the care plan before the outcome is met. The student may need additional support and time to reach the outcome.

The charge nurse overhears a nurse state, "That patient is 78 years old—too old to learn how to change a dressing." How should the charge nurse respond? A. "Please don't impose your view of the patient's culture on them." B. "I wish you would try to demonstrate more cultural sensitivity." C. "Try to be open to your patient's culture, to make the biggest impact." D. "Grouping all older adults as having trouble learning is a form of stereotyping."

d. Stereotyping is assuming that all members of a group are alike. This is not an example of cultural competence nor is the nurse imposing her culture on the patient. The information in this scenario does not suggest the nurse is not open to her patient's culture.

A nurse incorporates the "five values that epitomize the caring professional nurse" (identified by the American Association of Colleges of Nursing) into their home health care practice. Which attribute best reflects agreement with the code of ethics and accepted standards of practice? A. Altruism B. Autonomy C. Human dignity D. Integrity

d. The American Association of Colleges of Nursing defines integrity as acting in accordance with an appropriate code of ethics and accepted standards of practice. Altruism is a concern for the welfare and well-being of others. Autonomy is the right to self-determination, and human dignity is respect for the inherent worth and uniqueness of individuals and populations.

A nurse is formulating a clinical question using the PICOT format. The nurse begins with determining the "P," which focuses on which element? A. Comparison to another similar protocol B. Clearly defined, focused literature review of procedures C. Specific identification of the purpose of the study D. Explicit descriptions of the population of interest

d. The P in the PICOT format represents an explicit description of the patient population of interest. I represents the intervention, C represents the comparison, O stands for the outcome, and T stands for the time.

A nurse enters a patient's room and finds them vomiting bright red blood. After taking vital signs, the nurse communicates the event to the health care provider using the SBAR format. Which information will the nurse include in the "A" portion of the SBAR communication? Exhibit: Electronic health record (EHR) Past medical history Peptic ulcer, Bleeding disorder Vital Signs T 98.8°F, P 111, RR 20, BP 98/50, Pulse oximetry 96% A. Admitted with peptic ulcer and bleeding disorder B. Found vomiting in bathroom C. Anti-ulcer medication recommendation D. Vital signs, oxygen saturation, bright red emesis

d. The SBAR method is used to improve hand-off communication. SBAR, which stands for Situation, Background, Assessment, and Recommendations, provides a clear, structured, and easy to use framework. Vital signs, oxygen saturation, and the presence of emesis and its color are assessments.

The charge nurse overhears an AP yelling loudly to a patient who is hard of hearing, while transferring them from the bed to a chair. Upon entering the room, which response by the charge nurse is most appropriate? A. "Please speak more quietly so you don't disturb the other patients." B. "Let me help you with your transfer technique." C. "When you are finished, be sure to apologize for shouting." D. "When your patient is safe and comfortable, meet me at the desk."

d. The charge nurse should direct the AP to see to the patient's safety, then address any concerns privately. The nurse then can discuss appropriate use of therapeutic communication.

A nurse working in an "Aging in Place" facility interviews a married couple in their late 70s. Based on Duvall's Developmental Tasks of Families, what developmental task is most appropriate for the nurse to assess? A. Maintenance of a supportive home base B. Strength of the marital relationship C. Ability to cope with loss of energy and privacy D. Adjustment to retirement years

d. The developmental tasks of the family with older adults are to adjust to retirement and possibly to adjust to the loss of a spouse and loss of independent living. Maintaining a supportive home base and strengthening marital relationships are tasks of the family with adolescents and young adults. Coping with loss of energy and privacy is a task of the family with children.

A nurse is writing an evaluative statement for a patient who is trying to lower cholesterol through diet and exercise. Which evaluative statement is most correctly written? A. "Outcome met." B. "1/21/25—Patient reports no change in diet." C. "Outcome not met. Patient reports no change in diet or activity level." D. "1/21/25—Outcome met. Cholesterol level has decreased 10 mg."

d. The evaluative statement should contain a date; the words "outcome met," "outcome partially met," or "outcome not met"; and the patient data or behaviors that support this decision. The other answer choices are incomplete statements.

A nurse is caring for a patient with an eye infection with a moderate amount of discharge. What is the most appropriate technique for the nurse to use when cleansing this patient's eyes? A. Using diluted hydrogen peroxide on a clean washcloth to wipe the eyes B. Wiping the eye from the outer canthus toward the inner canthus C. Positioning the patient on the opposite side of the eye to be cleansed D. Cleansing the eye using a different section of the cloth for each stroke until clean

d. The nurse applies gloves for the cleaning procedure, uses water or normal saline, and a clean washcloth or gauze to cleanse the eyes. After dampening a cleaning cloth with the solution of choice, the nurse wipes once while moving from the inner canthus to the outer canthus of the eye to reduce forcing debris into the area drained by the nasolacrimal duct. The nurse should turn the cleansing cloth and use a different section for each stroke until the eye is clean.

A surgeon will not attempt a life-saving repair of a ruptured aneurysm unless the patient agrees to receive blood transfusions. Although receiving blood products is against the patient's religious beliefs, the surgeon ordered four units of packed red blood cells. What action will the nurse take first? A. Administer the blood transfusion B. Call the patient's family and ask them to reason with the patient C. Discuss obtaining a court order to save the patient's life D. Maintain the patient's comfort and support their decision

d. The nurse does not force patients to participate in care that conflicts with their values. Imposing such care may engender feelings of guilt and alienation from a religious or cultural group and create a threat to the patient's well-being.

A nurse enters a patient's room and examines the patient's intravenous (IV) fluids and cardiac monitor. When asked, "who are you?", which response by the nurse is most appropriate? A. "I'm just the IV therapist checking your IV." B. "I've been transferred to this division and will be caring for you." C. "I'm sorry, my name is John Smith and I am your nurse." D. "I am John Smith, your nurse, and I'll be caring for you until 11 PM."

d. The nurse should identify themselves, ensure the patient knows what will be happening, and the duration of their relationship.

The nurse is assigned to care for a group of patients. Which patient will the nurse assess first? A. Postoperative patient reporting pain 4/10 B. Individual with pneumonia whose WBCs are now 7,000 C. Adolescent with a burn to the face who is going home tomorrow D. Patient's pulse oximetry reading 89%, as reported by AP

d. The nurse uses Maslow's hierarchy of needs to prioritize assessing the patient with hypoxemia, manifested by a pulse oximetry reading of 89%. The postoperative patient is reporting moderate pain; the patient with pneumonia has normal WBCs; the adolescent patient with a burn to the face is stable for discharge. These patients can be seen as soon as possible.

Two nurses are repositioning a patient and pulling the patient up in bed. Which of these steps is most appropriate to prevent injury to the nurses? A. Telling the patient to cross their arms and legs B. Pulling the patient from underneath the axilla toward the top of the bed C. Avoiding using a draw sheet to lift or reposition the patient D. Ensuring the bed is at the level of the nurses' hips E. Facing the head of the bed and rocking in synchrony

d. The nurses should face the direction the patient will move and rock in synchrony prior to moving the patient in that direction. A lifting or repositioning sheet or device is used to decrease friction and facilitate movement. While the patient can cross their arms, they can also be instructed to press their feet into the mattress to assist movement. The bed should be at the level of the nurses' elbows.

A new graduate nurse phones the surgeon to report their patient is having severe incisional pain. The surgeon asks about vital signs and appearance of the wound, causing the nurse to return to the bedside for additional assessments. Upon reflection with the preceptor, which characteristic of the nursing process should the nurse have remembered? A. Centric B. Dynamic C. Interpersonal D. Systematic

d. The nursing process is systematic, iterative, and overlapping. By reporting an isolated symptom, the nurse has overlooked the benefit of systematic and inclusive assessment. While the nursing process is presented as an orderly progression of phases, there is a dynamic interaction and flow of phases into one another.

When a nurse enters the patient's room to begin a nursing history, the nurse notes the patient's spouse is present. After greeting them, what action will the nurse take? A. Thank the spouse for being present B. Ask the spouse if they want to remain C. Ask the spouse to leave D. Ask the patient if they would like the spouse to stay

d. The patient has the right to privacy and to determine who will be present during the nursing history and exam. The nurse does not presume the patient's preference, as the decision belongs to the patient, not their spouse.

Determine the patient's risk for pressure injury using the Braden scale based on information in the (EHR). EHR 1430 Admission Assessment: S: Patient admitted from nursing home for sepsis, confusion, ambulatory dysfunction. B: 87-year-old patient, with history of heart failure and hypertension; comes to ED with shortness of breath and yellow sputum. A: Lungs with crackles, pale, short of breath on exertion, pulse oximetry 88%, skin fragile. Bedrest maintained. States has not eaten nor drank fluids for last 36 hours; incontinent of small amount of urine × 2. Responding to painful stimuli, not participating in turning or care. R: Need orders for oxygen, sputum culture, activity level. Consider IV fluids. J. Smith RN. A. No risk B. Moderate risk C. High risk D. Very high risk

d. The patient is at very high risk for pressure injury. This patient responds only to painful stimulate (1); is occasionally moist (3); is bedridden (1); has not eaten (1), and requires maximum assistance for moving (1) for a total of 7 points. The Braden scale scoring is: a score of 19 to 23 indicates no risk; 15 to 18, mild risk; 13 to 14, moderate risk; 10 to 12, high risk; and 9 or lower, very high risk. In addition, nurses use clinical judgment to incorporate risk factors and/or other health problems into preventative interventions.

A hospice nurse is caring for a patient with end-stage cancer. Which action demonstrates the nurse's commitment to promoting the patient's autonomy? A. Competently administering pain medication B. Giving undivided attention when listening to patient concerns C. Keeping a promise to obtain a counselor D. Supporting the patient in obtaining a durable power of attorney

d. The principle of autonomy obligates nurses to provide information and support patients' and their surrogates' need to make decisions that advance their interests. Acting with justice means giving each person their due, acting with fidelity involves keeping promises to patients, and acting with nonmaleficence means avoiding doing harm to patients.

A nurse is caring for a patient who is incontinent of stool and has developed a stage 3 pressure wound on the buttocks. What intervention will the nurse set as the priority of care? A. Increasing nutrition B. Promoting mobility C. Managing chronic pain D. Preventing infection

d. The priority in this situation is to prevent infection through contamination of the wound by stool. The other actions may be taken as needed, after infection prevention is addressed.

Which statement reflects a clear evaluation of the patient's primary problem? (EHR) 8:00 AM Admission note S. Patient with profound wheezing, tachycardia, and anxiety B. Patient has history of asthma, for which she regularly uses inhalers and carries a rescue inhaler A. Pulse oximetry 89%, cyanosis of lips, dyspnea with increased work of breathing R. Admit to telemetry unit, add IV corticosteroids and mini-nebulizer treatments A. The patient states they were terrified when they were fighting to breathe and the wheezing would not stop. B. The nurse determines the patient's strengths include adherence to their medication regimen. C. The care plan includes the health problem of impaired gas exchange, etiology, bronchospasm. D. At 10:00 AM, no wheezing on auscultation, pulse oximetry is 94%, the patient reports no anxiety; the outcome has been met.

d. The two-part evaluative statement includes a decision about how well the outcome was met, along with patient data or behaviors that support this decision. Outcomes may have been met, partially met, or not met. The nurse collects evaluative data to measure outcome achievement. While this may justify terminating the care plan, that is not necessarily so. Data to assess health problems and patient variables are collected during the first step of the nursing process.

A nurse enters the room of a patient with cancer. The patient is crying and states, "I feel so alone." How will the nurse best communicate a therapeutic response? A. The nurse stands at the patient's bedside and states, "I understand how you feel. My mother said the same thing when she was ill." B. The nurse places a hand on the patient's arm and states, "You feel so alone." C. The nurse stands in the patient's room and asks, "Why do you feel so alone? Your wife has been here every day." D. The nurse holds the patient's hand and asks, "Tell me what feeling so alone is like for you?"

d. The use of touch conveys acceptance, and the implementation of an open-ended question allows the patient time to verbalize freely.

A nurse on the rehabilitation unit is counseling a young adult athlete who sustained a traumatic below-the-knee amputation following a motorcycle accident. The patient refuses to eat or ambulate, stating, "What's the point? My life is over. I'll never be the football player I dreamed of becoming." What is the nurse counselor's best response? A. "You're young and have your whole life ahead of you. You should focus on your rehabilitation and make something of your life." B. "I understand how you must feel. I wanted to be a famous singer, but I wasn't born with the talent to be successful at it." C. "You should concentrate on other sports that you could play even with prosthesis." D. "I understand this is difficult for you. Would you like to talk about it now or would you prefer me to make a referral to someone else?"

d. This answer communicates respect and sensitivity to the patient's needs and offers an opportunity to discuss their feelings with the nurse or another health care professional. The other answers do not allow the patient to express their feelings and receive the counseling they need.

The parent of a young school-age child wants them to learn about healthy dietary choices related to diabetes. Which method of value transmission would be most helpful? A. Depriving the child of their favorite toy when they consume foods not on their diet B. Lecturing the child on the merits of healthy and unhealthy food choices C. Allowing the child to experiment and discuss the outcomes on their blood glucose D. Offering healthy meals and snacks and acting as a role model for healthful eating

d. Through modeling by observing parents, peers, and significant others, children learn what is of high or low value. When punishment is used to transmit values, children create negative associations with those values. Using lecturing and moralizing modes of value transmission, usually taught by parents or an institution (e.g., church or school), offers little opportunity for the child to weigh different values. Using a laissez-faire approach to value transmission, where no single set of values is presented as best, is not appropriate for a young child; this approach could permit permanent consequences of diabetes to develop.

A nurse is developing education for nurses and UAPs related to prevention of pressure injuries for residents in a long-term care facility. Which action to prevent pressure injury will the nurses delegate to the UAP? A. Maintaining the head of the bed elevated consistently B. Massaging over bony prominences C. Repositioning bedbound patients every 4 hours D. Using a mild cleansing agent when cleansing the skin

d. To prevent pressure injuries, the nurse teaches the UAP to cleanse the skin routinely and whenever soiling occurs by using a mild cleansing agent with minimal friction, and avoiding hot water. The nurse educates the UAP to minimize the effects of shearing force by limiting the amount of time the head of the bed is elevated, when possible. Bony prominences should not be massaged, and bed-bound patients should be repositioned every 2 hours.

A toddler with vomiting, diarrhea, and dehydration is being seen at an acute care center. During the admission interview, what question will the nurse ask the parents to elicit the most useful information? A. "Watching your child vomiting and in discomfort must have been scary." B. "This started yesterday, correct?" C. "Has this child has had anything to drink?" D. Could you tell me the color and approximate amount of the vomiting?

d. Using a clarifying question or comment allows the nurse to gain an understanding of the parents' observations, avoiding misunderstandings that could lead to an inappropriate nursing diagnosis. A reflective question technique involves repeating what the person has said or describes the person's feelings. Assertive questions are direct, demonstrating the ability to stand up for self or others, using open and honest communication. Open-ended questions encourage free verbalization and expression of what the parents believe to be true.

A nurse is caring for a patient who has been admitted the second time this month for hypertensive emergency. The care plan contains the health problem: Nonadherence Etiology: lack of knowledge of purpose of medications Signs and symptoms: BP, 220/112; readmitted for hypertensive crisis after 2 weeks When meeting the patient, which action will the nurse take first? A. Teach the patient that nonadherence may lead to stroke and heart disease B. Discuss what will motivate the patient to adhere to the medication regimen C. Explain that these medications are essential to their health and illness prevention D. Determine the patient's knowledge about the medications and their side effects

d. Using the nursing process, the nurse first assesses the patient's knowledge base; this also confirms the accuracy of the problem statement. Problem statements with unclear etiologies may lead to inappropriate, erroneous, or unhelpful interventions. If the patient has difficulty affording medications or is experiencing side effects, a collaborative problem can be resolved jointly by the nurse, social worker, and health care provider.

At 8 AM, a nurse receives a prescription for an analgesic to be administered every 4 hours PRN. The nurse plans to administer this medication at what times? A. 0800, 1200, 1600, 2000, 0000, 0400 hours B. Around the clock on even hours C. Six times daily D. Upon patient request, within prescribed time intervals

d. When administering PRN medication, the patient receives medication when it is requested or required, and when the clinical parameters of the order, including timing between doses, are met. PRN orders are commonly written for the treatment of symptoms, in this instance, pain.

A nursing student is prioritizing interventions for a patient with diabetes who needs diagnostic testing, dressing changes, meal planning and counseling, and assistance with hygiene. The patient states, "I must have my hair washed before I can do anything else; I'm ashamed of the way I look." How will the student best prioritize this patient's care? A. Explain to the patient that there is not enough time to wash their hair today because of the busy schedule B. Schedule the testing and meal planning first and complete hygiene as time permits C. Perform the dressing changes first, schedule the testing and counseling, and complete hygiene last D. Wash the patient's hair and perform hygiene, schedule testing and counseling, then change the dressing

d. When time constraints and safety permit, priorities identified by the patient as most important are completed first. Washing the patient's hair and assisting with hygiene put the patient first, setting the tone for an effective nurse-patient partnership.

A nurse is looking for trends in a postoperative patient's vital signs. In which part of the electronic health record will the nurse find this information? A. Admission sheet B. Admission nursing assessment C. Progress notes D. Graphic record

d. While one set of vital signs may appear on the admission nursing assessment, the best place to find sequential recordings demonstrating a pattern or trend is the graphic record. The admission sheet and flow sheet do not include ongoing vital sign documentation.

A patient who injured the spine in a motorcycle accident is receiving rehabilitation services in a short-term rehabilitation center. The nurse caring for the patient tells the AP not to place the patient in which position? A. Side-lying B. Fowler's C. Sims' D. Prone

d. While placing the patient in the prone position for 30 minutes two or three times daily helps prevent knee and hip flexion contractures, it is contraindicated in patients who have spinal problems. The pull of gravity on the trunk when the patient lies prone produces a marked lordosis or forward curvature of the lumbar spine.

A nurse is using the classic elements of evaluation when caring for patients. Place the steps of evaluation in the proper order they are carried out. A. Interpreting and summarizing findings B. Collecting data to determine whether evaluative criteria and standards are met C. Documenting your judgment D. Terminating, continuing, or modifying the plan E. Identifying evaluative criteria and standards (i.e., expected patient outcomes)

e, b, a, c, d. The five classic elements of evaluation in order are (1) identifying evaluative criteria and standards (what the nurse looks for during evaluation, that is, expected patient outcomes); (2) collecting data to determine whether these criteria and standards are met; (3) interpreting and summarizing findings; (4) documenting your judgment; and (5) terminating, continuing, or modifying the plan.


Related study sets

Adobe Photoshop CC Lesson 3 - Questions

View Set

Chapter 14 Warm-up & Flexibility

View Set

Chapter 9 - Penalties & Other Tax Issues

View Set

Ch. 10 - Muscle Tissue and Organization pt. 1

View Set

AIDS Fundamentals (PUBLIC HEALTH PORTION)

View Set

Chapter 7 Nursing Management of Pain During Labor and Birth

View Set

Chapter 17 Section 2 Quiz Review

View Set

College Bio Mindtaps chapters 2 & 3

View Set